Thyroid, Parathyroid, Adrenal Flashcards

1
Q

A 40-year-old male presents to your office after noticing a nodule on his thyroid gland. He has no associated symptoms and his thyroid studies are normal. An ultrasound shows a 1cm solid mass that is aspirated with a fine-needle. The results are indeterminate. What is the next BEST step in management?

A. Repeat thyroid function tests

B. Follow up in three months

C. Core needle biopsy

D. Lobectomy

E. Total thyroidectomy

A

B. Follow up in three months

May observe since small or may repeat if inconclusive if indeterminate; more likely benign

How well did you know this?
1
Not at all
2
3
4
5
Perfectly
2
Q

What is the most common histologic variant of thyroid cancer?

A. Follicular

B. Hurthle

C. Medullary

D. Papillary

A

D. Papillary

How well did you know this?
1
Not at all
2
3
4
5
Perfectly
3
Q

A 47-year-old female presents with a 1.5cm thyroid nodule. Work-up shows follicular cells on FNA biopsy. What is the next BEST step?

A. Observation

B. Repeat FNA

C. Core needle biopsy

D. Thyroid lobectomy

E. Total thyroidectomy

A

D. Thyroid lobectomy

How well did you know this?
1
Not at all
2
3
4
5
Perfectly
4
Q

A 33-year-old female presents with heat intolerance, sweating, weight loss, palpitations, and proptosis. Which of the following is the MOST likely underlying cause of her symptoms?

A. Autoantibodies

B. Elevated levels of catecholamines

C. Elevated glucagon

D. Viral infection

E. Hyperfunctioning adenoma

A

A. Autoantibodies

How well did you know this?
1
Not at all
2
3
4
5
Perfectly
5
Q

A 31-year-old female has been treated for Grave’s disease with PTU for the last month with minimal improvement in symptoms. She asks about other treatment options for her disease. All of the following are indications for a thyroidectomy for Grave’s disease except:

A. Presence of a thyroid nodule

B. Failed medical management

C. Obstructive goiter

D. Severe thyrotoxicosis

E. Thyroid storm

A

E. Thyroid storm

How well did you know this?
1
Not at all
2
3
4
5
Perfectly
6
Q

A 42-year-old female scheduled to have a thyroidectomy for refractory hyperthyroidism. The week before her scheduled procedure, she developed cellulitis after scraping her arm. She now presents at the ER with fever, diaphoresis, and tachycardia to 130-140s. She appears to be extremely restless and confused. All of the following are accepted treatments for this patient, EXCEPT:

A. Preoperative Lugol’s solution

B. Propanolol

C. Dexamethasone

D. Phenoxybenzamine

A

D. Phenoxybenzamine

For pheochromocytoma

How well did you know this?
1
Not at all
2
3
4
5
Perfectly
7
Q

A 31-year-old female presents to your office complaining of a several week history of heat intolerance, palpitations, and diarrhea. On exam, her upper eyelids appear retracted. Her laboratory values show elevations in T3 and T4, a low TSH, and the presence of thyroid antibodies. The following are contraindications for the use of RAI to treat this patient’s condition, EXCEPT:

A. Pregnancy

B. Lactating

C. Presence of thyroid nodule

D. Exophthalmos

E. Mild severity of disease

A

E. Mild severity of disease

How well did you know this?
1
Not at all
2
3
4
5
Perfectly
8
Q

A 58-year-old male, who is employed as an auctioneer, is seen in clinic two weeks after his total thyroidectomy. He returned to work two days ago and complains that, although his voice is clear, he now requires the use of a megaphone, and still loses his voice before the end of the day. Which of the following is TRUE?

A. Injury to the external branch of the superior laryngeal nerve is less common than injury to the recurrent laryngeal nerve (RLN)

B. The treatment is vocal exercises and surgical alignment of the affected cord to the midline position if necessary

C. The nerve injured in this patient is normally found lateral to the superior pole of the thyroid

D. The muscle innervated by the injured nerve controls motion of the larynx, affecting the voice quality without airway compromise

E. The nerve is parallel to the superior thyroid vessels

A

D. The muscle innervated by the injured nerve controls motion of the larynx, affecting the voice quality without airway compromise

How well did you know this?
1
Not at all
2
3
4
5
Perfectly
9
Q

A 32-year-old female has an FNA performed on a 1cm thyroid nodule that was reported as a well differentiated papillary carcinoma. Which of the following criteria yields the BEST prognosis for this patient?

A. Age

B. Absence of capsular invasion

C. Absence of metastatic disease

D. Size of the nodule

E. Histology

A

A. Age

How well did you know this?
1
Not at all
2
3
4
5
Perfectly
10
Q

A 25-year-old female in her 12th week of pregnancy presents to her PCP with a thyroid nodule she noticed at home. After complete workup, she is diagnosed with papillary thyroid cancer. Which of the following is the MOST appropriate treatment in this scenario?

A. Thyroidectomy in second trimester

B. High doses of propylthiouracil

C. Thyroidectomy followed by I-131 prior to delivery

D. Thyroidectomy 6 weeks following delivery

A

A. Thyroidectomy in second trimester

How well did you know this?
1
Not at all
2
3
4
5
Perfectly
11
Q

A newborn child with a known family history of medullary thyroid cancer is found to be positive for the RET proto-oncogene. What is the BEST management for this patient?

A. Check calcitonin levels every 6 months

B. Annual fine needle aspiration of the thyroid

C. Annual ultrasound

D. Thyroidectomy by age 10

E. Thyroidectomy with central node dissection by age 2

A

E. Thyroidectomy with central node dissection by age 2

How well did you know this?
1
Not at all
2
3
4
5
Perfectly
12
Q

A 43-year-old female presents to your office with a 1cm thyroid nodule. Fine needle aspiration (FNA) confirms malignancy. The remaining thyroid gland appears normal. She has elevated calcitonin levels. What is the MOST appropriate management?

A. I-131 therapy

B. Right thyroid lobectomy

C. Total thyroidectomy

D. Total thyroidectomy with level VI lymph node dissection

E. Total thyroidectomy with right lateral neck dissection

A

D. Total thyroidectomy with level VI lymph node dissection

Medullary–> has to include central node dissection

How well did you know this?
1
Not at all
2
3
4
5
Perfectly
13
Q

An 18-year old man was recently seen by his PCP for a neck mass. An FNA biopsy was performed. The pathology was consistent with medullary thyroid cancer. All of the following are indicated in this patient’s condition EXCEPT:

A. CT scan of the abdomen and pelvis

B. 24s urine metanephrines

C. MRI brain

D. Serum calcium levels

A

C. MRI brain

How well did you know this?
1
Not at all
2
3
4
5
Perfectly
14
Q

A patient is recently diagnosed with MEN I syndrome after finding hyperparathyroidism, a gastrinoma, and a prolactinoma. What is the BEST next step in management?

A. Bromocriptine

B. Trans-sphenoid adenoma resection

C. Four gland parathyroidectomy with autoimplantation

D. Parathyroid adenoma resection

E. Enucleation of pancreatic mass

A

C. Four gland parathyroidectomy with autoimplantation

MEN 1: PPP

MEN 2A: Med, Pheo, Para

MEN 2B: Med, Pheo…

How well did you know this?
1
Not at all
2
3
4
5
Perfectly
15
Q

Which of the following illustrates a similarity between MEN 1 and MEN 2 syndromes?

A. Need to correct pheochromocytoma first in both

B. Gastrinoma is the most common pancreatic mass

C. Same genetic defect

D. Both need four gland parathyroidectomy

E. Thyroid cancer is the most common cause of death in both

A

D. Both need four gland parathyroidectomy

How well did you know this?
1
Not at all
2
3
4
5
Perfectly
16
Q

When performing a parathyroidectomy for hyperparathyroidism, you identify two enlarged glands which you remove. You obtain the following parathyroid hormone (PTH) values:

Prior to skin incision: 94 pg/mL
Prior to gland #1 excision: 344 pg/mL
5 min after excision of gland #1: 286 pg/mL
10 min after excision of gland #1: 279 pg/mL
Prior to gland #2 excision: 266 pg/mL
5 min after excision of gland #2: 114 pg/mL
10 min after excision of gland #2: 58 pg/mL

Which of the following is true?

A. Gland #1 was not hypersecreting

B. You must continue to search for hypersecreting glands until a 10 minute post-excision PTH value falls below 47 pg/mL

C. There is no need for further excision at this point

D. With successful removal of hyperfunctioning glands, the patient becomes eucalcemic and needs no further testing

E. If a hyperfunctioning gland has been identified preoperatively by sestamibi scan, then intra-op PTH values are not necessary

A

C. There is no need for further excision at this point

How well did you know this?
1
Not at all
2
3
4
5
Perfectly
17
Q

Which of the following illustrates a similarity between MEN 1 and MEN 2 syndromes?

A. Need to correct pheochromocytoma first in both

B. Gastrinoma is the most common pancreatic mass

C. Same genetic defect

D. Both need four gland parathyroidectomy

E. Thyroid cancer is the most common cause of death in both

A

D. Both need four gland parathyroidectomy

How well did you know this?
1
Not at all
2
3
4
5
Perfectly
18
Q

When performing a parathyroidectomy for hyperparathyroidism, you identify two enlarged glands which you remove. You obtain the following parathyroid hormone (PTH) values:

Prior to skin incision: 94 pg/mL
Prior to gland #1 excision: 344 pg/mL
5 min after excision of gland #1: 286 pg/mL
10 min after excision of gland #1: 279 pg/mL
Prior to gland #2 excision: 266 pg/mL
5 min after excision of gland #2: 114 pg/mL
10 min after excision of gland #2: 58 pg/mL

Which of the following is true?

A. Gland #1 was not hypersecreting

B. You must continue to search for hypersecreting glands until a 10 minute post-excision PTH value falls below 47 pg/mL

C. There is no need for further excision at this point

D. With successful removal of hyperfunctioning glands, the patient becomes eucalcemic and needs no further testing

E. If a hyperfunctioning gland has been identified preoperatively by sestamibi scan, then intra-op PTH values are not necessary

A

C. There is no need for further excision at this point

How well did you know this?
1
Not at all
2
3
4
5
Perfectly
19
Q

A 42-year-old female presents to your office complaining of muscle weakness and bone pain. A workup reveals an elevated serum calcium and PTH, along with elevated urine calcium. A sestamibi scan is performed which shows mildly increased uptake in the right inferior parathyroid gland. The patient is taken to the operating room and the rifgt inferior gland is resected and sent to pathology. The frozen pathology report described hyperplasia of the gland. What is the next BEST step in management of this patient?

A. Close the incision

B. Inspect the superior gland for adenoma

C. Inspect the contralateral side for adenoma

D. Biopsy the other glands

E. Total parathyroidectomy with autoimplantation

A

E. Total parathyroidectomy with autoimplantation

How well did you know this?
1
Not at all
2
3
4
5
Perfectly
20
Q

Following a parathyroidectomy, a 62-year-old man has persistent hyperparathyroidism. Where is the most common location for an ectopic parathyroid gland?

A. Thymus

B. Anatomical excision

C. Carotid sheath

D. Posterior portion of the tongue

A

A. Thymus

How well did you know this?
1
Not at all
2
3
4
5
Perfectly
21
Q

A 64-year-old female develops renal failure secondary to hypertension and develops secondary hyperparathyroidism to a kidney transplant, but continues to have hypercalcemia and symptoms of bone pain. What is the BEST next step in management?

A. IV fluids and diuretics

B. Sestamibi scan

C. Parathyroid adenectomy

D. Subtotal parathyroidectomy

E. Kidney transplant

A

D. Subtotal parathyroidectomy

How well did you know this?
1
Not at all
2
3
4
5
Perfectly
22
Q

A 50-year-old female comes to your office complaining of a one month history of weakness and fatigue. On exam, the patient has a palpable neck mass. Laboratory data shows an elevated PTH and a serum calcium level of 15. What is the BEST choice in management for this patient?

A. FNA of palpable mass

B. Sestamibi scan with parathyroid adenectomy

C. Total parathyroidectomy with autoimplantation

D. En bloc total parathyroidectomy with ipsilateral thyroidectomy

E. Neoadjuvant chemotherapy with en bloc total parathyroidectomy

A

D. En bloc total parathyroidectomy with ipsilateral thyroidectomy

How well did you know this?
1
Not at all
2
3
4
5
Perfectly
23
Q

What is the best diagnostic exam for thyroid lesions?

A

TSH (most sensitive)

How well did you know this?
1
Not at all
2
3
4
5
Perfectly
24
Q

What is the best physical exam parameter for thyroid lesions?

A

Heart rate (take sleeping heart rate; if >90, consider hyperthyroidism)

How well did you know this?
1
Not at all
2
3
4
5
Perfectly
25
Q

What are general treatment options for thyroid lesions?

A

Medical: Best for pregnant patients

RAI: Cannot be used in pregnant patients because it will cross the placenta and ablate the fetal thyroid tissue.

Surgery: Leave 2g of thyroid tissue, best for Graves’ disease.

How well did you know this?
1
Not at all
2
3
4
5
Perfectly
26
Q

What are the usual differentials for thyroid lesions?

A

INFLAMMATORY (Thyroiditis)

1) Acute suppurative thyroiditis
- Pain, tenderness, fever, dysphagia
- Tx: Drainage of abscess (sometimes needing thyroidectomy) + antibiotics

2) Chronic thyroiditis
- Most common: Hashimoto’s (autoimmune, antibodies against thyroid tissue, diffuse enlargement of gland, risk of developing papillary carcinoma)

NONMALIGNANT (adenoma, colloid goiter)

MALIGNANT
1) Well-differentiated
- Papillary (most common); and
Follicular (usually anterior neck mass with scalp mass)
- Comes from a single cell (follicular cells)
- Confirmed through FNAB; for results coming out as follicular tumors or neoplasm, consider them as non-specific as they could be an adenoma, colloid goiter, or cancer.
- Main difference is METASTATIC SPREAD:
—> Papillary mets: Lymphatic
—> Follicular mets: Hematogenous

2) Medullary thyroid CA
- Familial, associated with MEN IIA/IIB
- Check sx in parathyroid and pancreas
- Arise from C cells or parafollicular cells
- Calcitonin is used as a tumor marker
- May spread through lymphatics.
- Tx: Total thyroidectomy, neck dissection

3) Anaplastic thyroid CA (rapid growth, death from asphyxia or compression)
4) Lymphoma (least common, check other nodal basins– inguinal, axillary, thoracic, abdominal, tx with chemo)

How well did you know this?
1
Not at all
2
3
4
5
Perfectly
27
Q

What are principles in staging thyroid cancer?

A

All anaplastic cancers are stage IV lesions (in thyroid CA, stage IV does NOT necessarily mean distant mets, which is Stage IVC).

Staging of the neck is just N0 or N1, meaning presence or absence indicating good prognosis.

Staging of well-differentiated thyroid CA will depend on the age.

For patients <55yo, there is just Stage I and II. For patients >55yo, you have the usual staging (Stage I-IV).

After staging the patient, determine if low or high risk for recurrence using the following classification schemes:
A - age
M - mets
E - extent
S - size

A - age
G - grade
E - extent
S - size

Age cut-off: 55
Size cut-off: 4cm

Positive for any makes it high risk. Implication would be on prognosis and extent of surgery– if low risk, do a subtotal thyroidectomy (involving isthmus and pyramidal lobe). If high-risk, do a total thyroidectomy.

For the neck:
Papillary CA: Neck dissection if N1
Follicular CA: None

In general, if there are no clinically palpable nodes (N0), no neck dissection is warranted. However, if a total thyroidectomy is warranted, you should do a central node compartment dissection.

For distant mets of follicular CA:

  • Surgery (if possible, rarely done)
  • RAI (after doing RAI scan to check if there is uptake of RAI in distant mets)
  • Radiotherapy (if there is no RAI uptake during scanning)
How well did you know this?
1
Not at all
2
3
4
5
Perfectly
28
Q

Discuss RAI.

A

RAI Scan
Diagnostic, NOT therapeutic, used just to check for presence or absence of distant mets and if it is responsive to it (3 mci dosage used).

RAI Ablation
Done to kill remaining thyroid tissue in high-risk patients wherein only a subtotal thyroidectomy is performed (30 mci).

RAI Treatment/Therapy
For distant metastases (100-200 mci).

Timing of giving RAI is 6 weeks, time when TSH is at its highest level.

After treating distant mets, start Hormonal Suppressive Therapy, thyroid hormone dosage should be at levels that would bring TSH level below the minimal normal requirement.

Thyroglobulin can be used as a hormone marker, because they are produced by the thyroid follicles.

How well did you know this?
1
Not at all
2
3
4
5
Perfectly
29
Q

Primary hyperparathyroidism is due to?

A

1) Hyperplasia
2) Adenoma
3) Carcinoma

How well did you know this?
1
Not at all
2
3
4
5
Perfectly
30
Q

How is primary hyperparathyroidism treated?

A

1) Hyperplasia
Total parathyroidectomy with autotransplantation, or removal of the 3 1/2 of the glands.

2) Adenoma
Removal of diseased gland and biopsy of the rest.

3) Carcinoma
Total parathyroidectomy most of the time, including thyroid lobe.

How well did you know this?
1
Not at all
2
3
4
5
Perfectly
31
Q

How is secondary hyperparathyroidism treated?

A

Secondary is due to renal or bone problems; address cause.

How well did you know this?
1
Not at all
2
3
4
5
Perfectly
32
Q

A 44-year-old female presents to the emergency room complaining of left sided flank pain. A CT scan shows an obstructing kidney stone. The patient notes that she has a history of multiple kidney stones on both sides. Further workup on the patient finds elevated serum PTH and calcium, and elevated urine calcium. Which of the following is the BEST next step in management of this patient?

a. Sestamibi scan
b. Bilateral neck dissection and adenoma removal
c. 3 ½ gland parathyroidectomy
d. Total parathyroidectomy with autotransplantation into the forearm

A

b. Bilateral neck dissection and adenoma removal

How well did you know this?
1
Not at all
2
3
4
5
Perfectly
33
Q

What embryologic structures does the thyroid originate from?

A

The medial thyroid comes from the first and second pharyngeal pouches.

Lateral portions of the thyroid and parafollicular C cells arise from the fourth and fifth pharyngeal pouches.

How well did you know this?
1
Not at all
2
3
4
5
Perfectly
34
Q

What is the arterial supply of the thyroid?

A

Superior thyroid artery (from external carotid artery) Inferior thyroid artery (from thyrocervical trunk)

How well did you know this?
1
Not at all
2
3
4
5
Perfectly
35
Q

What is the venous drainage of the thyroid?

A

Superior and middle thyroid veins (drain into internal jugular vein)
Inferior thyroid veins (drain into innominate and brachiocephalic veins)

How well did you know this?
1
Not at all
2
3
4
5
Perfectly
36
Q

The recurrent laryngeal nerve innervates all of the muscles of the larynx except this muscle:

A

Cricothyroid muscle, which is innervated by the external branch of the superior laryngeal nerve

How well did you know this?
1
Not at all
2
3
4
5
Perfectly
37
Q

What structures do the recurrent laryngeal nerves wrap around?

A

Subclavian artery on the right

Arch of the aorta on the left

How well did you know this?
1
Not at all
2
3
4
5
Perfectly
38
Q

Injury to which nerve results in loss of projection and voice fatigability?

A

Superior laryngeal nerve

How well did you know this?
1
Not at all
2
3
4
5
Perfectly
39
Q

What is the mechanism of action of propylthiouracil (PTU) and methimazole?

A

Both drugs block peroxidase oxidation of iodide to iodine, thereby inhibiting incorporation of iodine into T4 and T3.

PTU also inhibits peripheral conversion of T4 to T3.

How well did you know this?
1
Not at all
2
3
4
5
Perfectly
40
Q

Which drug crosses the placenta: PTU or methimazole?

A

Methimazole

PTU is the safer choice in pregnancy.

How well did you know this?
1
Not at all
2
3
4
5
Perfectly
41
Q

When steroids are given in severe or acute hyperthyroid conditions, how do they work?

A

Steroids inhibit peripheral conversion of T4 to T3 and lower serum TSH by suppressing the pituitary-thyroid axis.

How well did you know this?
1
Not at all
2
3
4
5
Perfectly
42
Q

What is the Wolff-Chaikoff effect?

A

Large doses of iodine given after an antithyroid medication can inhibit thyroid hormone release by disrupting the coupling of iodide.

This is a transient effect.

How well did you know this?
1
Not at all
2
3
4
5
Perfectly
43
Q

What are the 3 most commons cause of hyperthyroidism?

A

Graves disease,
toxic multinodular goiter, and
toxic adenoma

How well did you know this?
1
Not at all
2
3
4
5
Perfectly
44
Q

What is the etiology of Graves disease?

A

Autoantibodies to TSH receptors (also called thyroid-stimulating antibodies or TSAb) bind and stimulate thyroid hormone production.

This leads to thyrotoxicosis, diffuse goiter, pretibial myxedema, and proptosis.

How well did you know this?
1
Not at all
2
3
4
5
Perfectly
45
Q

What is the treatment for Graves disease?

A

Antithyroid medication
Thyroid ablation
With/without thyroidectomy

How well did you know this?
1
Not at all
2
3
4
5
Perfectly
46
Q

A 55-year-old woman presents with a 3-year history offatigue and mild, diffuse, nontender thyroid enlargement, and 15-lb weight loss. What is the most likely diagnosis?

A

Hashimoto thyroiditis

How well did you know this?
1
Not at all
2
3
4
5
Perfectly
47
Q

What is the most common cause of hypothyroidism in adults?

A

Hashimoto thyroiditis

How well did you know this?
1
Not at all
2
3
4
5
Perfectly
48
Q

What is the first-line treatment for Hashimoto thyroiditis?

A

Thyroid hormone replacement

How well did you know this?
1
Not at all
2
3
4
5
Perfectly
49
Q

What will pathology show in a patient with Hashimoto thyroiditis?

A

Lymphocytic infiltrate

How well did you know this?
1
Not at all
2
3
4
5
Perfectly
50
Q

A 35-year-old female presents with sudden onset ofsevere pain and associated swelling and tenderness of her thyroid with fever, chills, and dysphagia following an acute upper respiratory infection. What is the most likely diagnosis?

A

Acute suppurative thyroiditis

How well did you know this?
1
Not at all
2
3
4
5
Perfectly
51
Q

What is the treatment for acute suppurative thyroiditis?

A

Antibiotics

Occasionally, abscess drainage

How well did you know this?
1
Not at all
2
3
4
5
Perfectly
52
Q

A 35-year-old female presents with moderate swelling and tenderness of her thyroid with repeated exacerbations and remissions over several months following an acute upper respiratory infection. What is the most likely diagnosis?

A

Subacute (de Quervain) thyroiditis

How well did you know this?
1
Not at all
2
3
4
5
Perfectly
53
Q

What is the treatment for subacute (de Quervain) thyroiditis?

A

NSAIDs, steroids

How well did you know this?
1
Not at all
2
3
4
5
Perfectly
54
Q

A 40-year-old female presents with hypothyroidism and symptoms of tracheal and esophageal compression and is found to have dense fibrosis throughout her thyroid gland. What is the most likely diagnosis?

A

Riedel fibrous struma
Painless, progressive goiter
Usually euthyroid but may become hypothyroid

How well did you know this?
1
Not at all
2
3
4
5
Perfectly
55
Q

What is the treatment for Riedel fibrous struma?

A

Thyroid hormone replacement and steroids

Surgery may be necessary to relieve obstructive symptoms

How well did you know this?
1
Not at all
2
3
4
5
Perfectly
56
Q

What is the treatment of thyroid storm?

A

PTU or methimazole every 4 to 6 hours and inorganic iodide to block synthesis and release of thyroid hormones
Dexamethasone to inhibit peripheral conversion of T4 to T3
Propranolol
Fever reduction
General resuscitation

How well did you know this?
1
Not at all
2
3
4
5
Perfectly
57
Q

What is the most common cause of thyroid enlargement?

A

Multinodular goiter

How well did you know this?
1
Not at all
2
3
4
5
Perfectly
58
Q

What are indications for surgery with a multinodular goiter?

A

Presence of or inability to rule out malignancy Compressive symptoms
Cosmetic deformity

How well did you know this?
1
Not at all
2
3
4
5
Perfectly
59
Q

What is the diagnostic test of choice for the evaluation of a thyroid nodule?

A

FNA
It has a sensitivity of 86% and
a specificity of 91%

How well did you know this?
1
Not at all
2
3
4
5
Perfectly
60
Q

What is usually the first manifestation of multiple endocrine neoplasia (MEN) 2a and2b?

A

Medullary thyroid carcinoma (MTC)

How well did you know this?
1
Not at all
2
3
4
5
Perfectly
61
Q

What are the cytologic features of MTC?

A

Amyloid among neoplastic cells
Immunohistochemistry positive for calcitonin
Positive staining for carcinoembryonic antigen (CEA) or calcitonin gene-related peptide
Heterogeneity with polygonal or spindle-shaped cells

How well did you know this?
1
Not at all
2
3
4
5
Perfectly
62
Q

What percentage of MTC is sporadic?

A

75% to 80%
Unable to differentiate familial versus sporadic at presentation– all should be tested for RET mutation, pheochromocytoma, and hyperparathyroidism

How well did you know this?
1
Not at all
2
3
4
5
Perfectly
63
Q

What operation should be done for MTC?

A

Total thyroidectomy

High incidence of multicentric disease

How well did you know this?
1
Not at all
2
3
4
5
Perfectly
64
Q

By what age should MEN 2a and MEN 2b patients undergo total thyroidectomy?

A

MEN2a: Age 6

MEN 2b: Age 1 to 2

How well did you know this?
1
Not at all
2
3
4
5
Perfectly
65
Q

What tumor markers are measured in MTC?

A

Calcitonin and CEA

How well did you know this?
1
Not at all
2
3
4
5
Perfectly
66
Q

In MEN2, if both pheochromocytoma and MTC are present, which should be treated first?

A

Pheochromocytoma

How well did you know this?
1
Not at all
2
3
4
5
Perfectly
67
Q

What cytologic features are indicative of anaplastic thyroid carcinoma?

A

Grossly firm and white

Marked heterogeneity with spindle, polygonal, or multinucleated cells

How well did you know this?
1
Not at all
2
3
4
5
Perfectly
68
Q

What is the prognosis for anaplastic thyroid carcinoma?

A

Poor: only few survive more than 6 months

How well did you know this?
1
Not at all
2
3
4
5
Perfectly
69
Q

Who typically gets anaplastic thyroid carcinoma?

A

Elderly patients with a long-standing goiter

How well did you know this?
1
Not at all
2
3
4
5
Perfectly
70
Q

What is the treatment for a small anaplastic thyroid carcinoma?

A

Total thyroidectomy with or without external beam radiation

Has small improvement in survival, especially for younger patients

How well did you know this?
1
Not at all
2
3
4
5
Perfectly
71
Q

What is the treatment for anaplastic thyroid carcinoma with compressive symptoms?

A

Debulking surgery with tracheostomy

How well did you know this?
1
Not at all
2
3
4
5
Perfectly
72
Q

Which patient has a higher likelihood of malignancy: the patient with a solid thyroid lesion versus cystic thyroid lesion?

A

Solid thyroid lesion

How well did you know this?
1
Not at all
2
3
4
5
Perfectly
73
Q

Which patient has a higher likelihood of malignancy: the patient with a solitary thyroid lesion versus multiple thyroid lesions?

A

Solitary thyroid lesion

How well did you know this?
1
Not at all
2
3
4
5
Perfectly
74
Q

Which patient has a higher likelihood of malignancy: the patient with a hot thyroid lesion versus cold thyroid lesion?

A

Cold thyroid lesion

How well did you know this?
1
Not at all
2
3
4
5
Perfectly
75
Q

Which type is the most common thyroid cancer and has the best prognosis?

A

Papillary thyroid cancer, about 70% to 80% of all thyroid cancers

How well did you know this?
1
Not at all
2
3
4
5
Perfectly
76
Q

Which subtypes carry a worse prognosis?

A

Insular Columnar Tall cell

How well did you know this?
1
Not at all
2
3
4
5
Perfectly
77
Q

What are the histologic findings for papillary thyroid carcinoma?

A

Psammoma bodies

Orphan Annie nuclei

How well did you know this?
1
Not at all
2
3
4
5
Perfectly
78
Q

What is the treatment for papillary thyroid carcinoma?

A

High-risk, large (>2 cm), or bilateral tumors:
Total thyroidectomy

Low-risk, small (<1 cm), or unilateral tumors:
Thyroid lobectomy and isthmusectomy

How well did you know this?
1
Not at all
2
3
4
5
Perfectly
79
Q

What must be done prior to thyroidectomy for suspected or proven malignancy to assess for lymph node involvement?

A

Neck ultrasound

How well did you know this?
1
Not at all
2
3
4
5
Perfectly
80
Q

What laboratory test is followed after surgery to monitor recurrence?

A

Thyroglobulin

How well did you know this?
1
Not at all
2
3
4
5
Perfectly
81
Q

True or False: Positive cervical nodes affect the prognosis of papillary thyroid carcinoma.

A

True. Lymph node involvement indicates more aggressive disease.

How well did you know this?
1
Not at all
2
3
4
5
Perfectly
82
Q

What histologic findings are needed to define malignancy in follicular cancer?

A

Vascular or capsular invasion

How well did you know this?
1
Not at all
2
3
4
5
Perfectly
83
Q

What is the most common site of distant metastasis for follicular thyroid carcinoma?

A

Bone
Next most common is lung
Spread is hematogenous

How well did you know this?
1
Not at all
2
3
4
5
Perfectly
84
Q

Which has a worse prognosis: Hurthle cell carcinoma or follicular carcinoma?

A

Hurthle cell carcinoma

Higher recurrence rate usually to regional lymph nodes

How well did you know this?
1
Not at all
2
3
4
5
Perfectly
85
Q

What is the tumor, node, metastasis (TNM) stage for a 57-year-old patient who underwent a total thyroidectomy for a 2.5-cm mass that was determined to be papillary thyroid carcinoma?

A

All lymph nodes were free of disease, and there was no extrathyroidal disease.

This patient has T2NOMO disease.

Because the patient is older than 45 years, this is stage II papillary thyroid cancer.

How well did you know this?
1
Not at all
2
3
4
5
Perfectly
86
Q

What are the indications for I-131 therapy?

A

1 to 4 cm with aggressive histology, >4 cm

Extrathyroid extension, vascular invasion, multifocal disease, lymph node involvement, distant metastasis

How well did you know this?
1
Not at all
2
3
4
5
Perfectly
87
Q

What is the significance of BRAF mutation in thyroid cancer?

A

Increased mortality and disease recurrence

How well did you know this?
1
Not at all
2
3
4
5
Perfectly
88
Q

What structure are the superior parathyroid glands embryologically derived from?

A

Fourth branchial pouch

How well did you know this?
1
Not at all
2
3
4
5
Perfectly
89
Q

What structure are the inferior parathyroid glands embryologically derived from?

A

Third branchial pouch

How well did you know this?
1
Not at all
2
3
4
5
Perfectly
90
Q

What structure is the thymus embryologically derived from?

A

Third branchial pouch

How well did you know this?
1
Not at all
2
3
4
5
Perfectly
91
Q

What is the arterial blood supply to the superior parathyroid glands?

A

Inferior thyroid artery (occasionally by branches ofthe superior thyroid artery)

How well did you know this?
1
Not at all
2
3
4
5
Perfectly
92
Q

What is the arterial blood supply to the inferior parathyroid glands?

A

Inferior thyroid artery

How well did you know this?
1
Not at all
2
3
4
5
Perfectly
93
Q

What is the spatial relationship of the inferior parathyroid gland to the recurrent laryngeal nerve and inferior thyroid artery?

A

Inferior parathyroid glands are medial to the recurrent laryngeal nerves and located below the inferior thyroid artery

How well did you know this?
1
Not at all
2
3
4
5
Perfectly
94
Q

What is the spatial relationship of the superior parathyroid gland to the recurrent laryngeal nerve and inferior thyroid artery?

A

Superior parathyroid glands are lateral to the recurrent laryngeal nerves and located above the inferior thyroid artery

How well did you know this?
1
Not at all
2
3
4
5
Perfectly
95
Q

Describe the mechanisms by which PTH increases serum calcium concentration:

A

Bone: enhances resorption of bone matrix by osteoclasts

Kidney: increases tubular reabsorption of filtered calcium and decreases tubular reabsorption of fIltered phosphate

Intestine: stimulates renal vitamin D complex synthesis, which increases intestinal absorption of calcium (indirect effect)

How well did you know this?
1
Not at all
2
3
4
5
Perfectly
96
Q

What laboratory test is the most sensitive and specific way to diagnose hyperparathyroidism?

A

Intact parathyroid hormone level (elevated in >95% of patients with primary hyperparathyroidism)

How well did you know this?
1
Not at all
2
3
4
5
Perfectly
97
Q

What is the half-life of parathyroid hormone?

A

2 to 4 minutes

How well did you know this?
1
Not at all
2
3
4
5
Perfectly
98
Q

What is the desired decline in the intraoperative parathyroid hormone assay that confirms that the suspected abnormal parathyroid tissue was resected?

A

50% decrease from baseline PTH or a drop of the PTH to the normal range

How well did you know this?
1
Not at all
2
3
4
5
Perfectly
99
Q

Describe the process of vitamin D formation and activation:

A

7-dehydrocholesterol undergoes ultraviolet activation to form vitamin D (cholecalciferol) –>

hydroxylated in the liver to form 25-hydroxycholecalciferol–>

undergoes a second hydroxylation in the kidneys to its most active form 1,25-dihydroxycholecalciferol

How well did you know this?
1
Not at all
2
3
4
5
Perfectly
100
Q

What cells secrete the hormone calcitonin?

A

Parafollicular (C cells) of the thyroid

How well did you know this?
1
Not at all
2
3
4
5
Perfectly
101
Q

Which type(s) of hyperparathyroidism (primary, secondary, or tertiary) is associated with high serum PTH levels and high-normal to elevated serum calcium levels?

A

Primary and tertiary hyperparathyroidism

How well did you know this?
1
Not at all
2
3
4
5
Perfectly
102
Q

Which type of hyperparathyroidism is associated with high serum PTH levels and low-normal to low serum calcium levels?

A

Secondary hyperparathyroidism

How well did you know this?
1
Not at all
2
3
4
5
Perfectly
103
Q

Which type of hyperparathyroidism is considered a compensatory response of the parathyroid glands to hypocalcemia?

A

Secondary hyperparathyroidism

How well did you know this?
1
Not at all
2
3
4
5
Perfectly
104
Q

Which type of hyperparathyroidism occurs when long-standing stimulation of the parathyroid glands by hypocalcemia results in autonomous hyperfunctioning of the parathyroid glands?

A

Tertiary hyperparathyroidism

How well did you know this?
1
Not at all
2
3
4
5
Perfectly
105
Q

What acid-base disturbance can be seen with primary hyperparathyroidism?

A

Hyperchloremic metabolic acidosis

How well did you know this?
1
Not at all
2
3
4
5
Perfectly
106
Q

What chloride:phosphate ratio is highly suggestive of primary hyperparathyroidism?

A

Chloride:phosphate ratio > 33 is highly suggestive of primary hyperparathyroidism

How well did you know this?
1
Not at all
2
3
4
5
Perfectly
107
Q

What is the most common cause primary hyperparathyroidism?

A
Single adenoma (80%)
Diffuse hyperplasia and multiple adenomas can also occur
How well did you know this?
1
Not at all
2
3
4
5
Perfectly
108
Q

What oncogene increases the risk for a parathyroid adenoma?

A

PRAD-1

How well did you know this?
1
Not at all
2
3
4
5
Perfectly
109
Q

What are the classic gross findings diagnostic of a parathyroid adenoma?

A

Single enlarged gland with 3 normal or small remaining glands associated with the histologic finding of hyperplastic tissue

How well did you know this?
1
Not at all
2
3
4
5
Perfectly
110
Q

Indications for initial parathyroidectomy for a patient with primary hyperparathyroidism:

A

Symptomatic:
Typical bone, renal, gastrointestinal, neuromuscular symptoms

Asymptomatic:
Significant or life-threatening hypercalcemia
Presence of kidney stones detected by abdominal imaging
Medical surveillance not desirable/possible
Serum calcium level > 1 mg/dL above upper limit of normal
Age <50 years
Osteoporosis (bone density T-score < -2.5 at any site)
Creatinine clearance decreased by >30%
Elevated 24-hour urinary calcium excretion (>400 mg/d)

How well did you know this?
1
Not at all
2
3
4
5
Perfectly
111
Q

What is the difference between persistent and recurrent hyperparathyroidism?

A

Persistent refers to hypercalcemia that remains within 6 months of initial surgery

Recurrent refers to hypercalcemia that returns after 6 months of initial surgery

How well did you know this?
1
Not at all
2
3
4
5
Perfectly
112
Q

What is the most common cause of persistent hyperparathyroidism?

A

Missed adenoma

How well did you know this?
1
Not at all
2
3
4
5
Perfectly
113
Q

What are the causes for recurrent hyperparathyroidism?

A

New adenoma formation or recurrent parathyroid cancer

How well did you know this?
1
Not at all
2
3
4
5
Perfectly
114
Q

Indications for reoperative parathyroidectomy for persistent or recurrent disease:

A
Ongoing nephrolithiasis
Worsening renal function
Worsening bone disease as evidenced by bone mineral density scores 
Associated neuromuscular symptoms
Associated psychiatric symptoms
Worrisome progressive hypercalcemia
How well did you know this?
1
Not at all
2
3
4
5
Perfectly
115
Q

Indications for parathyroideciomy in patients with secondary hyperparathyroidism:

A

Development of open ulcerative skin lesions from calcinosis or calciphylaxis
Persistent bone pain or pathologic fractures (renal osteodystrophy)
Ectopic calcifications
Intractable pruritus
Worsening disease with failure of medical management

How well did you know this?
1
Not at all
2
3
4
5
Perfectly
116
Q

General intraoperative algorithm for searching a “missing” parathyroid gland:

A

Open and inspect the thyroid capsule, palpate gland.

Consider intraoperative ultrasound.

Dissect superior thymic/paratracheal tissue and complete a cervical thymectomy for missing inferior parathyroid glands
Mobilize the pharynx and esophagus to look in the parapharyngeal/retropharyngeal/esophageal spaces for missing superior glands.

Open the carotid sheath and expose the common carotid and inspect for potential parathyroid glands.

Ligate the ipsilateral inferior thyroid artery and/or perform a thyroid lobectomy (record the location ofall confirmed glands identified).

End the procedure and follow the patient for any evidence ofpersistent hypercalcemia.

Can reimage the patient for evidence of ectopic parathyroid adenoma.

How well did you know this?
1
Not at all
2
3
4
5
Perfectly
117
Q

What is the most common location for a “missing” parathyroid gland at reoperation?

A

Normal anatomic position.

How well did you know this?
1
Not at all
2
3
4
5
Perfectly
118
Q

Generally when will cryopreserved parathyroid tissue have the best viability?

A

Within the first 2 years after freezing

How well did you know this?
1
Not at all
2
3
4
5
Perfectly
119
Q

How is parathyroid tissue autotransplanted?

A

The resected parathyroid tissue is minced into 1-mm fragments.

A pocket is then made in the desired muscle.

Ten to 20 1-mm fragments of minced parathyroid tissue are inserted into the muscle pocket, which is closed and tagged with hemoclips/suture (for future identification}

How well did you know this?
1
Not at all
2
3
4
5
Perfectly
120
Q

Why is the forearm used preferentially over the sternocleidomastoid for autotransplantation of parathyroid tissue?

A

Easier to re-explore if patient develops persistent or recurrent disease from autotransplanted tissue.

Easier to identify PTH gradients with peripheral blood draws in forearm versus neck (can place BP cuff).

How well did you know this?
1
Not at all
2
3
4
5
Perfectly
121
Q

Why do some patients experience postoperative hypocalcemia?

A

Bone hunger,
hypomagnesemia,
failure of parathyroid remnant or graft

How well did you know this?
1
Not at all
2
3
4
5
Perfectly
122
Q

What imaging study is used for preoperative localization for reoperative parathyroid surgery and for minimally invasive parathyroideciomy?

A

Technetium-sestamibi scan, ultrasound, CT-sestamibi fusion, or 4D CT

How well did you know this?
1
Not at all
2
3
4
5
Perfectly
123
Q

In general, what is the most common cause of hypoparathyroidism?

A

Surgical trauma to parathyroid glands during thyroid or parathyroid exploration

How well did you know this?
1
Not at all
2
3
4
5
Perfectly
124
Q

What operation is performed for parathyroid carcinoma?

A

En bloc tumor resection that includes ipsilateral thyroid lobectomy and resection of adjacent soft tissues

How well did you know this?
1
Not at all
2
3
4
5
Perfectly
125
Q

True or False: A frozen section biopsy for suspected parathyroid cancer should be performed before surgical excision.

A

False; capsular rupture may occur with the potential of spreading tumor cells in the neck

How well did you know this?
1
Not at all
2
3
4
5
Perfectly
126
Q

What is the most common location for parathyroid cancer to metastasize?

A

Lung

How well did you know this?
1
Not at all
2
3
4
5
Perfectly
127
Q

What is the most common cause of hyperparathyroidism in MEN 1?

A

Parathyroid hyperplasia (90%)

How well did you know this?
1
Not at all
2
3
4
5
Perfectly
128
Q

Which of the following needs to be corrected first in MEN 1— hyperparathyroidism, gastrinoma, or prolactinoma?

A

Hyperparathyroidism; need to correct calcium first.

How well did you know this?
1
Not at all
2
3
4
5
Perfectly
129
Q

Which of the following needs to be corrected first in MEN 2— hyperparathyroidism, pheochromocytoma, or MTC?

A

Pheochromocytoma

How well did you know this?
1
Not at all
2
3
4
5
Perfectly
130
Q

What hormones are released from the posterior pituitary?

A

ADH and oxytocin

How well did you know this?
1
Not at all
2
3
4
5
Perfectly
131
Q

What hormones are released from the anterior pituitary?

A

ACTH, FSH, LH, GH, TSH, and prolactin

How well did you know this?
1
Not at all
2
3
4
5
Perfectly
132
Q

What gonadotropic hormone promotes spermatogenesis or ovarian follicle maturation?

A

FSH

How well did you know this?
1
Not at all
2
3
4
5
Perfectly
133
Q

What gonadotropic hormone promotes testicular testosterone production?

A

LH

How well did you know this?
1
Not at all
2
3
4
5
Perfectly
134
Q

What drug may be given as primary therapy in patients who are not operative candidates with excessive production of GH by a pituitary adenoma?

A

Octreotide {decreases serum levels of GH and the downstream growth factor somatomedin C)

How well did you know this?
1
Not at all
2
3
4
5
Perfectly
135
Q

What is the size cutoff to determine whether a pituitary lesion is a microadenoma versus macroadenoma?

A

Microadenoma <1 cm

Macroadenoma >1 cm

How well did you know this?
1
Not at all
2
3
4
5
Perfectly
136
Q

What imaging study is the gold standard for evaluating the pituitary?

A

MRI with gadolinium

How well did you know this?
1
Not at all
2
3
4
5
Perfectly
137
Q

What is the most common pituitary adenoma?

A

Prolactinoma

How well did you know this?
1
Not at all
2
3
4
5
Perfectly
138
Q

What is the treatment for prolactinoma?

A

Bromocriptine

Transsphenoidal resection for failure of medical management

How well did you know this?
1
Not at all
2
3
4
5
Perfectly
139
Q

Meningococcal sepsis/infection results in adrenal gland hemorrhage, leading to adrenal insufficiency:

A

Waterhouse-Friderichsen syndrome

How well did you know this?
1
Not at all
2
3
4
5
Perfectly
140
Q

Results from arachnoid herniation secondary to a congenital defect in the diaphragma seilae:

A

Empty sella syndrome (primary)

How well did you know this?
1
Not at all
2
3
4
5
Perfectly
141
Q

A 35-year-old female with 1-week postpartum history of placental hemorrhage who presents with trouble lactating and amenorrhea:

A

Sheehan syndrome

142
Q

Hyperpigmentation and pituitary enlargement (resulting in amenorrhea and visual problems) after bilateral adrenalectomy:

A

Nelson syndrome

143
Q

Calcified cyst that is remnant of Rathke pouch and can present with endocrine abnormalities, headache, hydrocephalus, and visual disturbances:

A

Craniopharyngioma

144
Q

What are signs/symptoms of Cushing syndrome?

A

Acne, buffalo hump, depression, diabetes, easy bruising, hirsutism, hypertension, moon fades, myopathy, purple striae, truncal obesity, weakness

145
Q

What enzyme is present almost exclusivelyin the adrenal medulla and organ of Zuckerkandl? What does it do?

A

Phenylethanolamine-N-methyltransferase (PNMT)

Converts norepinephrine to epinephrine via methylation

146
Q

What enzyme is involved in the rate-limiting step of catecholamine synthesis?

A

Tyrosine hydroxylase (hydroxylates tyrosine to dihydroxyphenylalanine [DOPA])

147
Q

What intermediate substrates are involved in the synthesis ofepinephrine from tyrosine?

A
Tyrosine ----> 
DOPA ----> 
Dopamine ----> 
Norepinephrine ---->
Epinephrine
148
Q

Which artery does the superior adrenal artery originate from?

A

Inferior phrenic artery

149
Q

Which artery does the middle adrenal artery originate from?

A

Aorta

150
Q

Which artery does the inferior adrenal artery originate from?

A

Renal artery

151
Q

The left adrenal vein empties into which vein?

A

Left renal vein

152
Q

The right adrenal vein empties into which vein?

A

Inferior vena cava

153
Q

What does the zona glomerulosa of the adrenal cortex produce?

A

Aldosterone

154
Q

What does the zona fasciculata of the adrenal cortex produce?

A

Glucocorticoids

155
Q

What does the zona reticularis of the adrenal cortex produce?

A

Androgens/estrogens

156
Q

What is the most common deficiency seen with congenital adrenal hyperplasia?

A

21-hydroxylase deficiency (90%)

157
Q

What are the indications for surgical resection of an adrenal mass?

A
4 cm or larger
enlarging {0.5cm over 6 months), 
functioning,
ominous characteristics (nonhomogeneous, poor washout) on imaging
158
Q

What laboratory tests should be ordered in the workup of an adrenal mass?

A
Plasma metanephrines, 
cortisol, 
BMP, 
aldosterone, 
renin, 
catecholamines
159
Q

Primary hyperaldosteronism accounts for what percentage of all cases of HTN?

A

1%

160
Q

Classic Conn syndrome is characterized by:

A

HTN, hypokalemia, polyuria

161
Q

What test can differentiate primary hyperaldosteronism from secondary hyperaldosteronism?

A

Plasma renin level (suppressed with primary and not suppressed with secondary)

162
Q

What is the #1 cause of primary hyperaldosteronism?

A

Adrenal adenoma

163
Q

What is the aldosterone:renin ratio that is suggestive of an aldosteronoma?

A

Greater than 30 is diagnostic for primary hyperaldosteronism

164
Q

What is the aldosterone suppression test?

A

Give a saline infusion; if the urinary aldosterone level is elevated (>12μg/d), the diagnosis of primary hyperaldosteronism is confirmed.

165
Q

What is the best imaging modality to distinguish aldosteronoma from bilateral hyperplasia?

A

CT scan of the abdomen with thin cuts (3-5 mm) through the adrenal glands

166
Q

What acid-base disturbance will be seen with primary hyperaldosteronism?

A

Hypochloremic metabolic alkalosis

167
Q

What test can be performed to lateralize an aldosterone-producing hyperfunctioning adrenal gland in the setting of adrenal hypertrophy, bilateral nodules, or absence of lesions?

A

Adrenal venous sampling (AVS)

168
Q

In which patients should AVS be performed?

A

Patients with diagnosed hyperaldosteronism greater than 40 years old

169
Q

What is the adrenal vein aldosterone-to-cortisol ratio required to lateralize an aldosteronoma/unilateral adrenal hyperplasia?

A

At least 5 times higher on the affected side; if ratios from both sides are similar, then suspect bilateral hyperplasia.

170
Q

If the captopril test is performed in a patient with an aldosterone-secreting tumor, what will happen to serum aldosterone levels?

A

No change.

If the patient has bilateral adrenal hyperplasia, a captopril test will cause decreased aldosterone levels.

171
Q

What competitive inhibitor of aldosterone is usually given to control HTN and normalize potassium levels in a patient with an aldosteronoma/unilateral hyperplasia before surgery?

A

Spironolactone

172
Q

What is the best treatment approach for an aldosteronoma or unilateral hyperplasia?

A

Laparoscopic adrenalectomy

173
Q

What is the treatment for hyperaldosteronism from adrenal hyperplasia?

A

Medical therapy with calcium channel blockers, potassium, and spironolactone.

If refractory hypokalemia, then bilateral adrenalectomy, steroids, and fludrocortisone.

174
Q

What is the most sensitive and specific test for making the diagnosis of Cushing syndrome?

A

24-hour urinary free cortisol

175
Q

What is the most common site of ectopic ACTH-producing tumor? Second most common site?

A
#1 - Small cell lung carcinoma
#2 - Carcinoid
176
Q

What ACTH level signifies an ACTH-independent cause of endogenous glucocorticoid hypersecretion?

A

<5 pg/mL (low)

177
Q

What ACTH level signifies an ACTH-dependent cause of endogenous glucocorticoid hypersecretion?

A

15 pg/mL (high)

178
Q

How is the low-dose overnight dexamethasone suppression test performed?

A

Administer dexamethasone 1 mg PO at 11 P.M.; check plasma cortisol at 8 A.M. the next day.

Cortisol level will be markedly suppressed (<1.8 μ.g/dL) in normal individuals.

If cortisol level is not suppressed, the patient has Cushing syndrome.

179
Q

How is the high-dose overnight dexamethasone suppression test performed?

A

Obtain baseline plasma cortisol;
administer dexamethasone 8 mg PO at 11 P.M.;
check 8 A.M. plasma cortisol.

Suppression occurs if there is >50% reduction in plasma cortisol, which indicates Cushing syndrome from a pituitary source (if not suppressed, then from an adrenal or ectopic cause).

180
Q

What happens to the ACTH level during a high-dose dexamethasone suppression test with a pituitary tumor?

A

ACTH level is suppressed

181
Q

What happens to the ACTH level during a high-dose dexamethasone suppression test with an ectopic source?

A

ACTH level is not suppressed

182
Q

How is the corticotropin-releasing hormone (CRH) stimulation test performed?

A

Check baseline serum ACTH and cortisol.

Give 1 mcg/kg CRH IV.

Draw serum ACTH and cortisol 15 minutes after administering CRH.

183
Q

How does the CRH stimulation test distinguish between pituitary versus adrenal tumor versus ectopic ACTH causes of endogenous glucocorticoid hypersecretion?

A

ACTH increases at a minimum of 35% above baseline with pituitary lesions only (others do not).

184
Q

What laboratory tests establish the diagnosis of a virilizing tumor?

A

Increased levels of serum dehydroepiandrostenedione (DHEA-S),
serum testosterone, and
24-hour urine 7-hydroxysteroids and 17-ketosteroids

185
Q

How does a dexamethasone suppression test differentiate between an adrenal versus ovarian cause for a virilizing tumor?

A

24-hour ketosteroids and serum androgens will be suppressed with ovarian tumors but not adrenal tumors

186
Q

What happens to gadolinium on MRI for a malignant adrenal tumor? For an adrenal adenoma?

A

Gadolinium persists in a malignant tumor (poor washout); gadolinium washes out rapidly from an adenoma (rapid washout)

187
Q

What is the #1 cause of adrenal insufficiency?

A

Withdrawal of exogenous steroids

188
Q

What electrolyte abnormalities are seen in adrenal insufficiency?

A

Hyponatremia, hyperkalemia

189
Q

What test can be performed to diagnose adrenal insufficiency?

A

ACTH stimulation test

190
Q

What is one of the first signs of the loss of hypothalamic regulation of cortisol secretion?

A

Loss of diurnal variation of cortisol levels

191
Q

What is the safest operation for patients with suspected adrenocortical carcinoma?

A

Open adrenalectomy

192
Q

What medication is commonly used for adjuvant therapy with residual, recurrent, or metastatic adrenocortical carcinoma?

A

Mitotane, a synthetic derivative of the insecticide dichlorodiphenyltrichloroethane (DDT) that directly suppresses the adrenal cortex and modifies the peripheral metabolism of steroids.

193
Q

What follow-up is needed for a patient with adrenal tumor <4 cm treated nonoperatively?

A

Repeat abdominal imaging every 6 months to a year and annual screening for cortisol and catecholamines/metanephrines for 5 years.

194
Q

What is the treatment for a patient with adrenal mass >4 cm?

A

Adrenalectomy; most likely malignant

195
Q

What disease process needs to be ruled out before performing a fine needle aspiration of a solitary adrenal mass?

A

Pheochromocytoma; inadvertent biopsy can be potentially lethal

196
Q

What laboratory test is the most sensitive for the detection of pheochromocytoma?

A

Plasma metanephrines (96%)

197
Q

What is the rule of 10s for pheochromocytoma?

A

10% bilateral, 10% malignant, 10% extra-adrenal, 10% hereditary, 10% in children

198
Q

In what ectopic sites can a pheochromocytoma be found?

A

Organ of Zuckerkandl, thorax/mediastinum, bladder, scrotum

199
Q

What familial syndromes are associated with pheochromocytoma?

A
MEN type 2, 
Sturge-Weber syndrome, 
tuberous sclerosis, 
von Hippel-Lindau disease, 
neurofibromatosis type I
200
Q

What is usually seen with a hypertensive “spell,” the hallmark clinical presentation of pheochromocytoma?

A

Diaphoresis,
headache,
palpitations, and
paroxysmal HTN

201
Q

True or False: Paragangliomas and pheochromocytomas located in the organ of Zuckerkandl can use PNMT to metabolize norepinephrine to epinephrine.

A

True; paragangliomas and pheochromocytomas in the organ of Zuckerkandl contain PNMT

202
Q

What medications should be given preoperatively for pheochromocytoma?

A

Phenoxybenzamine 10 mg BID and increase by 20 mg/d until BP and symptoms controlled;
add beta-blocker 3 days prior to surgery (Inderal 10 mg tid);
IVF hydration starting 2 days prior to surgery (typically patients are volume contracted)

203
Q

What rapid-acting agents should be available in the operating room when operating on a pheochromocytoma?

A

Neosynephrine, nitroprusside, lidocaine, esmolol, phentolamine

204
Q

What follow-up is recommended after adrenalectomy for pheochromocytoma?

A

Yearly 24-hour urinary metanephrines/catecholamines

205
Q

What is the localizing test of choice for adrenal pheochromocytoma?

A

CT scan of the abdomen with thin cuts (3-5 mm) through the adrenal glands

206
Q

What is the approximate percentage ofMEN 2 patients with pheochromocytoma?

A

50%

207
Q

What is the surgical treatment for pheochromocytoma in MEN 2?

A

Adrenalectomy for the affected side and observe the contralateral side until there is radiographic/biochemical evidence of disease.

Bilateral adrenalectomy for bilateral disease.

208
Q

What is the most common cancer to metastasize to the adrenal gland?

A

Breast cancer

209
Q

A 27-year-old woman presents to the office with a left-sided neck mass approximately 2.5 cm in size. FNA reveals papillary thyroid cancer. She has no palpable lymph nodes. What is the best management for this patient?

A. Left thyroid lobectomy with left modified radical neck dissection
B. Total thyroidectomy with bilateral modified radical neck dissection
C. Left thyroid lobectomy with postoperative I-131therapy
D. Total thyroidectomy with postoperative I-131 therapy
E. Total thyroidectomy alone

A

Answer: E.

For papillary thyroid cancers larger than 2 cm, total thyroidectomy is indicated.

Postoperative 1-131 therapy is given for tumors >4 cm or extrathyroidal disease.

Neck dissection is unnecessary without evidence of nodal disease.

210
Q

A 27-year-old woman presents to the office with a left-sided neck mass approximately 2.5 cm in size. FNA reveals papillary thyroid cancer. She has a 1-cm palpable left cervical lymph node. What is the best management for this patient?

A. Left thyroid lobectomy alone
B. Left thyroid lobectomy with left modified radical neck dissection
C. Total thyroidectomy with left modified radical neck dissection
D. Total thyroidectomy with postoperative I-131 therapy
E. Preoperative FNA of the palpable cervical lymph node

A

Answer: E.

The size of the tumor necessitates a total thyroidectomy.

This patient has a palpable cervical lymph node, and thus preoperative FNA of the lymph node should be done.

A lateral neck dissection is indicated if the lymph node is positive for malignancy.

211
Q

A 17-year-old male has a 2-cm right thyroid nodule. An FNA is done, and cells stain positive for calcitonin. Labs also reveal elevated calcium. What is the most likely diagnosis?

A. Familial MTC
B. Follicular thyroid cancer
C. MEN I
D. MEN 2a
E. MEN 2b
A

Answer: D.

Calcitonin-positive cells indicate a diagnosis of MTC. Elevated calcium suggests hyperparathyroidism.

Along with pheochromocytoma, these diagnoses make up MEN 2a.

MEN 2b would include MTC, pheochromocytoma, and mucosal neuromas.

212
Q

A 46-year-old woman presents to the emergency department with fever, tachycardia, diaphoresis, and tremors. She has a history of a goiter. Initial management diagnostic workup should include all of the following except:

A. PTU
B. Steroids 
C. Beta-blocker 
D. Oxygen 
E. I-131
A

Answer: E.

This patient has thyroid storm. Beta-blockers are used to control heart rate with inhibition of peripheral conversion of T4 to T3.

Steroids and PTU work by decreasing production and release of thyroid hormones.

IV fluids and oxygen are part of routine resuscitative care. I-131 has no role in the treatment of thyroid storm.

213
Q

Which ofthe following is NOT commonly seen in patients with MEN I syndrome?

A. Gastrinoma
B. Insulinoma
C. Prolactinoma
D. Parathyroidhyperplasia 
E. MTC
A

Answer: E.

MTC is seen in patients with MEN 2a and 2b.

Parathyroid hyperplasia is the earliest and most common manifestation of MEN 1. It develops in 80% to 100% of patients by age 40.

Gastrinomas are the most common pancreatic islet cell tumor in MEN 1, which occurs in about 50% ofpatients, followed by insulinomas, which develop in 10% to 15% of cases.

Prolactinomas are the most common pituitary lesion, which occurs in 10% to 50% of MEN 1 cases.

214
Q

What is the most common adrenal incidentaloma found on CT scan?

A. Adrenocorticalcarcinoma 
B. Cortical adenoma
C. Pheochromocytoma
D. Aldosteronoma
E. Adrenal cyst
A

Answer: B.

Nonfunctioning cortical adenomas comprise the majority of adrenal incidentalomas in patients without a history of cancer.

215
Q

Which of the following is an indication for surgery in a patient with asymptomatic primary hyperparathyroidism?

A. History of an episode of life-threatening hypercalcemia
B. Medical surveillance not desirable/possible
C. Age <50 years
D. Osteoporosis (bone density T-score

A

Answer: E.

The 2002 NIH consensus conference guidelines for parathyroidectomy in patients with asymptomatic primary hyperparathyroidism are:
serum calcium > 1 mg/dL above the upper limits ofnormal;
life-threatening hypercalcemia episode;
creatinine clearance reduced by 30%;
kidney stones on abdominal x-ray;
markedly elevated 24-hour urinary calcium excretion >400 mg/dL;
age <50 years;
long-term medical surveillance not desired or possible; and
substantially decreased bone mineral density at the lumbar spine, hip, or distal radius (T score

216
Q

A 65-year-old male is found to have an incidental right adrenal mass on CT scan during workup for trauma. He has no history of hypertension or malignancy. Physical examination is unremarkable. Blood work demonstrates normal electrolytes and plasma metanephrines. Urinary free cortisol is normal, and low-dose overnight dexamethasone suppression test shows a low cortisol level the following morning. The CT scan demonstrates a 6-cm irregularly shaped heterogeneous mass. What is the next appropriate step in management?

A. Perform CT-guided needle biopsy of adrenal mass
B. Repeat CT scan in 6 months
C. Perform open adrenalectomy
D. Give patient mitotane

A

Answer: C.

The patient is found to have an incidentaloma that is asymptomatic and nonfunctioning.

However, given its size of 6 cm and CT findings that suggest malignancy, the patient should undergo adrenalectomy.

Other CT scan features that suggest malignancy include evidence of necrosis, hemorrhage, local invasion, lymph node metastasis, and high attenuation.

Needle biopsy of an adrenal mass should only be performed in patients with a history of malignancy to rule out an adrenal metastasis after ruling out pheochromocytoma.

Mitotane is an adrenal cytotoxic agent that is used as adjuvant therapy for adrenocortical carcinoma.

217
Q

A 50-year-old woman presents with truncal obesity and hirsutism.

A 24-hour urinary free cortisol is markedly elevated. Low-dose dexamethasone suppression test fails to suppress plasma cortisol levels. Plasma ACTH levels are also elevated. High-dose dexamethasone suppression test yields low plasma cortisol levels. Which imaging study would most likely demonstrate the cause of her symptoms?

A. CT of the chest
B. CT of the abdomen
C. MRI of the sella turcica
D. Metaiodobenzylguanidine (MIBG} scan

A

Answer: C.

When Cushing syndrome is suspected, the first test should be a 24-hour urinary free cortisol.

If it is elevated, a low-dose dexamethasone suppression test is performed.

Failure to suppress cortisol levels indicates Cushing syndrome.

ACTH levels are then measured. Low ACTH levels indicate a cortisol-secreting tumor, such as a primary adrenal source.

A CT scan of the abdomen would be helpful in this case.

A high ACTH level suggests a pituitary or ectopic source of ACTH.

A high-dose dexamethasone suppression test will differentiate this.

The pituitary source will be suppressed; thus, pituitary MRI should be performed.

Failure to suppress production suggests an ectopic ACTH tumor, which most commonly is small cell lung cancer or bronchial tumors.

CT of the chest would be warranted in this case.

218
Q

A 55-year-old male with newly diagnosed hypertension complains of muscle weakness. Laboratory data demonstrate hypokalemia and low plasma renin activity. CT scan shows bilaterally enlarged adrenal glands without a definite mass. What is the next best step in management?

A. Give spironolactone
B. Perform selective venous catheterization
C. Perform unilateral adrenalectomy
D. Perform bilateral adrenalectomy

A

Answer: B.

The patient is suspected to have primary hyperaldosteronism, which may be due to bilateral adrenal hyperplasia or an aldosteronoma.

Selective venous catheterization for aldosterone sampling in adrenal veins can aid in localizing the aldosteronoma, thus determining ifsurgical intervention would be beneficial.

If the aldosteronoma is localized to 1 adrenal gland, then the patient should undergo unilateral adrenalectomy.

If bilateral adrenal glands are hyperfunctioning or if there is failure to localize the aldosteronoma, then medical management with spironolactone would be the mainstay therapy.

There is increased morbidity with bilateral resection, which is reserved for refractory hypokalemia for cases of bilateral hyperplasia.

219
Q

A 73-year-old female presents with hypertension and hypokalemia. Labs show elevated plasma aldosterone with an aldosterone-to-renin ratio of greater than 30. What are the next steps?

A. CT scan now and every 6 to 12 months
B. CT scan followed by adrenal venous sampling
C. CT scan and laparoscopic adrenalectomy if there is a unilateral mass
D. CT scan and bilateral adrenalectomy if there are bilateral masses

A

Answer: B.

The patient has Conn syndrome, or primary hyperaldosteronism. CT scan or imaging such as MRI should be done to evaluate for adrenal mass or hyperplasia.

At age greater than 40, AVS is also suggested due to the increased risk of incidental adrenal hyperplasia or mass.

This is recommended in the American Association of Clinical Endocrinologists and American Association ofEndocrine Surgeon guidelines for adrenal disease.

220
Q

A 66-year-old female presents with primary hyperparathyroidism. Her preoperative imaging suggests a right lower neck adenoma. An enlarged right inferior parathyroid adenoma was found and resected with intraoperative PTH drop from 220 to 150. What is the next operative step?

A. End the procedure and do further workup postoperatively
B. Proceed with 4-gland exploration
C. Right thyroid lobectomy
D. Identify and resect the right superior parathyroid gland then recheck PTH

A

Answer: B.

Intraoperative PTH testing indicates cure when the post-excision PTH decreases by at least 50% and to within normal range.

If PTH does not drop by at least 50% or remains elevated, 4-gland exploration should be pursued.

If a parathyroid gland cannot be located, first look in the expected anatomic positions, followed by the thymus, thyroid (using ultrasound), and within the carotid sheath.

221
Q

A 50-year-old male presents with a 3-an left adrenal mass found incidentally on imaging. On MRI, there is rapid washout of contrast from the mass. Cortisol, aldosterone, renin, and metanephrine levels are normal. What is the most appropriate treatment?

A. Laparoscopic left adrenalectomy
B. Open left adrenalectomy
C. Repeat imaging in 6 to 12 months followed by annual labs and imaging for 5 years 
D. Repeat imaging every 3 months
E. Mitotane
A

Answer: C.

Labs show no excess hormonal production, and rapid washout on MRI suggests a benign adrenal adenoma.

Follow-up imaging should be obtained in 6 to 12 months and annually along with labs for 5 years.

There is no indication of adrenalectomy or mitotane.

222
Q

A 72-year-old female presents with a 3-year history of worsening hypercalcemia. She has osteopenia (T-score -1.2), lethargy, muscle weakness, and urinary frequency. Her serum calcium level is elevated, and PTH level is at the upper limit of normal. 24-hour urine calcium was normal. What should be done to complete the workup and treat the disease?

A. Obtain vitamin D levels and supplement if low, then start cinacalcet
B. Obtain localization study and proceed with parathyroidectomy
C. Obtain an ultrasound and biopsy any suspected parathyroid adenoma
D. Repeat labs annually until she reaches NIH criteria for parathyroidectomy in asymptomatic patients

A

Answer: B.

The patient has mild to moderate symptoms of primary hyperparathyroidism.

Localization studies should be obtained in preparation for parathyroidectomy.

Biopsy of suspected parathyroid adenoma is not recommended.

In general, cinacalcet is indicated in severe primary hyperparathyroidism when the patient has a contraindication for surgery.

223
Q

A 45-year-old male with end-stage renal disease presents for consideration of parathyroidectomy for secondary hyperparathyroidism. What lab abnormalities are expected?

A. Vitamin D deficiency, hyperphosphatemia, hypocalcemia
B. Vitamin D deficiency, hypophosphatemia, hypercalcemia
C. Vitamin D deficiency, hyperphosphatemia, hypercalcemia
D. Vitamin D deficiency, hypophosphatemia, hypocalcemia

A

Answer: A.

Patients with end-stage renal disease and secondary hyperparathyroidism have vitamin D deficiency, hyperphosphatemia, and hypocalcemia.

They are treated with vitamin D supplementation and phosphate binders.

Calcimimetics such as cinacalcet are used for treatment of secondary hyperparathyroidism.

Parathyroidectomy is recommended when PTH is persistently greater than 800.

224
Q

A 32-year-old male is diagnosed with MENIIa syndrome. What is the BEST next step in management of the patient?

A. Adrenalectomy

B. Four gland parathyroidectomy with autoimplantation dissection

C. Parathyroid adenoma resection

D. Total thyroidectomy

E. Total thyroidectomy with central node

A

A. Adrenalectomy

225
Q

Which of the following illustrates a similarity between MEN1 and MEN2 syndromes?

A. Need to correct pheochromocytoma first in both

B. Gastrinoma is the most common pancreatic mass

C. Same genetic defect

D. Both need four gland parathyroidectomy

E. Thyroid cancer is the most common cause of death in both

A

D. Both need four gland parathyroidectomy

226
Q

When performing a parathyroidectomy for hyperparathyroidism, you identify two enlarged glands which you remove. You obtain the following PTH values:

Prior to skin incision: 94 pg/mL
Prior to gland #1 excision: 344 pg/mL
5 min after excision gland #1: 286 pg/mL
10 min after excision gland #1: 279pg/mL
Prior to gland #2 excision: 266 pg/mL
5 min after excision gland #2: 114 pg/mL
10 min after excision gland #2: 58 pg/mL

Which of the following is TRUE?

A. Gland #1 was not hypersecreting

B. You must continue to search for hypersecreting glands until a 10 minute post-excision PTH value falls below 47 pg/mL

C. There is no need for further excision at this point

D. With successful removal of hyperfunctioning glands, the patient becomes eucalcemic and needs no further testing.

E. If a hyperfunctioning gland has been identified preoperatively by sestamibi scan, then intraop PTH values are not necessary.

A

C. There is no need for further excision at this point

227
Q

A 42-year-old female presents to your office complaining of muscle weakness and bone pain. A workup reveals an elevated serum calcium and PTH, along with elevated urine calcium. A sestamibi scan is performed which shows mildly increased uptake in the right inferior parathyroid gland. The patient is taken to the operating room and the right inferior gland is resected and sent to pathology. The frozen pathology report describes hyperplasia of the gland. What is the next BEST step in management of this patient?

A. Close the incision

B. Inspect the superior gland for adenoma

C. Inspect the contralateral side for adenoma

D. Biopsy the other glands

E. Total parathyroidectomy with autoimplantation

A

E. Total parathyroidectomy with autoimplantation

228
Q

Following a parathyroidectomy, a 62-year-old man gas persistent hyperparathyroidism. Where is the MOST common location for an ectopic parathyroid gland?

A. Thymus

B. Anatomical location

C. Carotid sheath

D. Posterior portion of the tongue

A

A. Thymus

229
Q

A 64-year-old female develops renal failure secondary to hyperparathyroidism. She has a kidney transplant, but continues to have hypercalcemia and symptoms of bone pain. What is the next BEST step in managing this patient?

A. IV fluids and diuretics

B. Sestamibi scan

C. Parathyroid adenectomy

D. Subtotal parathyroidectomy

E. Kidney transplant

A

D. Subtotal parathyroidectomy

230
Q

A 50-year-old female comes to your office complaining of a one month history of weakness and fatigue. On exam the patient has a palpable neck mass. Laboratory data shows an elevated PTH and a serum calcium level of 15. What is the BEST choice in management for this patient?

A. FNA of palpable mass

B. Sestamibi scan with parathyroid adenectomy

C. Total parathyroidectomy with autoimplantation

D. En bloc total parathyroidectomy with ipsilateral thyroidectomy

E. Neoadjuvant chemotherapy with en bloc total parathyroidectomy

A

D. En bloc total parathyroidectomy with ipsilateral thyroidectomy

231
Q

Which surgeon was awarded the Nobel Prize in Physiol- ogy for Medicine for his work on the “physiology, pathol- ogy, and surgery of the thyroid gland?”

A. Theodore Billroth
B. Emil Kocher
C. John Hunter
D. Harvey Cushing

A

Answer:B

The Nobel Prize in Physiology or Medicine was awarded to Emil Kocher in 1909. In addition to his research on the physiology of the thyroid, Kocher’s operative methods greatly reduced the mortality risk of thyroidectomy.

The Kocher clamp was designed to prevent hemorrhage rom the hyper- vascular gland during thyroidectomy. (See Schwartz 10th ed., p. 1521.)

232
Q

What congenital anomaly arises from the formation of the thyroid gland?

A. The thyroid isthmus
B. The cricothyroid arch
C. A thyroglossal duct cyst
D. An endobronchial cyst

A

Answer: C

The medial thyroid anlage descends rom the base of the tongue through a channel called the thyroglossal duct at week 3 to 4 of gestation.

The duct normally closes after its descent, but may remain patent and is susceptible to secondary infection and dilatation, referred to as a thyroglossal duct cyst.

Removal is accomplished with the Sistrunk operation which also removes the central portion of the hyoid bone. (See Schwartz 10th ed., p. 1521.)

233
Q

The arterial supply of the thyroid arises from which of the following vessels?

A. The aorta

B. The external carotid arteries

C. The thyrocervical trunk

D. All of the above

A

Answer: D

The superior thyroid arteries arise from the external carotid arteries, and the inferior thyroid arteries arise rom the thyrocervical trunk shortly after their origin from the subclavian arteries.

A thyroid ima artery arises directly from the aorta or innominate artery in 1 to 4% of cases. (See Schwartz 10th ed., p. 1523.)

234
Q

In what location, relative to the inferior thyroid artery (ITA), is the recurrent laryngeal nerve (RLN) found?

A. Medial or posterior to the ITA
B. Lateral or anterior to the ITA
C. Passing between the branches of the ITA
D. All of the above

A

Answer: D

The RLN courses within the tracheoesophageal groove after emerging from the vagus nerve at the level of the aortic arch.

As it ascends in the neck, the recurrent laryngeal nerve (RNL) may branch, and may pass anterior, posterior, or interdigitate with branches of the ITA.

The location of the RLN must be confirmed before the ITA is divided. (See Schwartz 10th ed., p. 1524.)

235
Q

Although injury to the RLN results in hoarseness (unilat- eral injury) or airway obstruction (bilateral injury), injury to the superior laryngeal nerve (SLN) results in a more subtle injury, affecting the ability to

A. Speak loudly or sing high notes.
B. Cough.
C. Feel sensation in the anterior neck.
D. Grimace.

A

Answer: A

The external branch of the superior laryngeal nerve (SLN) lies on the inferior pharyngeal constrictor muscle and descends alongside the superior thyroid vessels before innervating the cricothyroid muscle.

Therefore the superior pole vessels should not be ligated en masse, but should be individually divided low on the thyroid gland.

Injury to the SLN leads to inability to tense the ipsilateral vocal cord, and impairs the ability to “hit high notes” while singing, or projecting the voice loudly. (See Schwartz 10th ed., p. 1524.)

236
Q

Thyroid hormones (T3 and T4) have regulatory roles in all of the following EXCEPT

A. The hypoxia and hypercapnia drives of the respiratory center in the brain.

B. Gastrointestinal motility.

C. The speed of muscle contraction and relaxation.

D. Visual acuity in low-light conditions (“night vision”).

A

Answer: D

Thyroid hormones are responsible for maintaining the normal hypoxic and hypercapnic drive in the respiratory center of the brain, and regulate gastrointestinal motility which leads to diarrhea in hyperthyroidism and constipation in hypothyroidism.

They also regulate bone and protein turnover and the speed of muscle contraction and regulation, hepatic gluconeogenesis, cholesterol synthesis, and intestinal glucose absorption. (See Schwartz 10th ed., p. 1528.)

237
Q

In North America, hyperthyroidism is most often caused by

A. Toxic multinodular goiter

B. Diffuse toxic goiter (Graves disease)

C. Thyroid cancer

D. Thyroid stimulating hormone-secreting pituitary adenoma

A

Answer: B

Graves disease, named after Robert Graves, the Irish physician who described the disorder in three patients in 1835, is the most common cause of hyperthyroidism in North America, and accounts or 60 to 80% of cases. (See Schwartz 10th ed., p. 1531.)

238
Q

Subtotal or total thyroidectomy is preferred for the treatment of Graves disease

A. When radioactive iodine therapy is contraindicated.

B. When the goiter is large or airway obstruction appears
imminent.

C. In patients with demonstrated poor compliance with anti-thyroid medications.

D. All of the above.

A

Answer: D

Subtotal or total thyroidectomy is now preferred over subtotal thyroidectomy due to a lower recurrence rate.

Surgery is preferred over medical therapy (radioactive iodine) in childbearing women who desire to have children in the near future, in noncompliant patients, or when airway obstruction appears likely.

(See Schwartz 10th ed., p. 1533.)

239
Q

What is the recommended course of action when fine needle aspiration biopsy (FNAB) of a thyroid nodule is “follicular neoplasm?”

A. Repeat FNAB

B. Lobectomy

C. Lobectomy and isthmusectomy

D. Total thyroidectomy

A

Answer: B

Follicular neoplasms of the thyroid are less aggressive than papillary neoplasms, and a fine needle aspiration biopsy (FNAB) may be unable to differentiate between a follicular adenoma and a follicular carcinoma.

For this reason unilateral lobectomy is recommended for this FNAB diagnosis.(See Schwartz 10th ed., p. 1539.)

240
Q

Which diseases are associated with germline mutations in the RET tyrosine kinase receptor gene?

A. Multiple endocrine neoplasia type 2A (MEN2A)

B. Multiple endocrine neoplasia type 2B (MEN2B)

C. Hirschsprung disease

D. All of the above

A

Answer: D

Mutations in the extracellular domain of the RET tyrosine kinase receptor are associated with multiple endocrine neoplasia type 2A (MEN2A), familial medullary thyroid cancer (FMTC), and Hirschsprung disease.

Mutations in the intracellular domain are associated with MEN2B, FMTC, and Hirschsprung disease.
(See Schwartz 10th ed)

241
Q

Children exposed to the Chernobyl disaster in 1986 subsequently demonstrated an increased incidence of which thyroid cancer?

A. Papillary thyroid cancer (PTC)

B. Follicular thyroid cancer (FTC)

C. Medullary thyroid cancer (MTC)

D. Anaplastic thyroid cancer (ATC)

A

Answer: A

Papillary thyroid cancer accounts for 80% of all thyroid malignancies and is the predominant thyroid cancer in children and individuals exposed to external radiation.
(See Schwartz 10th ed., p. 1542.)

242
Q

The recommended treatment or an otherwise healthy 50-year-old man with a 2-cm PTC in the left lobe diagnosed by FNAB is

A. Left lobectomy

B. Left lobectomy and isthmusectomy

C. Total left lobectomy and subtotal right lobectomy

D. Excisional biopsy with frozen section analysis

A

Answer:C

Total thyroidectomy or total lobectomy on the affected side with subtotal lobectomy on the non-affected side is the recommended treatment of choice for unifocal PTC greater than 1 cm in diameter.

Defiinitive operation can be performed without rozen section when the diagnosis is unequivocal on FNAB. (See Schwartz 10th ed., p. 1543.)

243
Q

An adolescent patient with a thyroid mass undergoes FNAB which returns as MCT. What other diseases should be screened for before treatment is undertaken?

A. Hyperparathyroidism

B. Pheochromocytoma

C. Mucocutaneous ganglioneuromas

D. All of the above

A

Answer: D

MC can be spontaneous (in 75%) or familial (in 25%) in MEN2.

MEN2A is associated with pheochromocytoma and hyperparathyroidism, whereas MEN2B is associated with pheochromocytoma, Marfanoid habitus, and mucocutaneous ganglioneuromas. (See Schwartz 10th ed., p. 1550.)

244
Q

An asymptomatic child with a normal physical examination is found to harbor a mutation in codon 918 of the RET tyrosine kinase receptor, compatible with MEN2B. Ultrasound of the neck is unremarkable and serum calcitonin levels are normal. What course is indicated?

A. Repeat examination and ultrasound yearly

B. Planned thyroidectomy in 3 to 5 years

C. Total thyroidectomy

D. Total thyroidectomy with bilateral neck dissection

A

Answer: C

Children with mutations at codon 634 of the RET tyrosine kinase receptor gene (MEN2A) are advised to undergo thyroidectomy before age 5, whereas children with mutations at codon 918 (MEN2B) should undergo thyroidectomy before age 1.

If ultrasound of the neck is normal and calcitonin levels are normal, a formal neck dissection can be avoided. (See Schwartz 10th ed., p. 1550.)

245
Q

Postoperative complications within 24 hours of thyroid surgery include

A. Hypocalcemia
B. Dyspnea
C. Dystonia
D. All of the above

A

Answer: D

Inadvertent injury (ischemia) or removal of the parathyroid glands can cause acute neuromuscular excitability due to hypocalcemia.

An expanding hematoma in the neck may not cause bleeding from the wound, but can compress the membranous portion of the trachea and cause dyspnea.

Nerve injuries can cause vocal cord paralysis or impaired speech. (See Schwartz 10th ed., p. 1556.)

246
Q

A patient with primary hyperparathyroidism undergoes neck exploration where our small, normal appearing glands are ound. What are the possible locations o an additional, supernumerary gland?

A. In the thyroid gland
B. In the thymus
C. In the tracheoesophageal groove
D. All of the above

A

Answer: D

Supernumerary parathyroid glands occur in 7 to 13% of people, and may be located in the thymus (most commonly), within the parenchyma of the thyroid gland, or in the tracheoesophageal groove, the mediastinum, or elsewhere in the neck. (See Schwartz 10th ed., p. 1557.)

247
Q

A 70-year-old woman with early dementia but otherwise good physical health has an elevated parathyroid hormone (PTH) level and a sestamibi scan which localizes a single focus of increased activity to the left lower neck. An ultrasound confrms an enlarged gland in the same area. What treatment is likely to provide the best outcome?

A. Bilateral neck exploration under general anesthesia.

B. Unilateral, “mini-incision” parathyroidectomy under
local anesthesia.

C. Minimally invasive videoscopic parathyroidectomy
rom a left axillary approach under general anesthesia.

D. Percutaneous alcohol ablation with ultrasound guidance under local anesthesia.

A

Answer: B

Localization studies such as sestamibi scans have been shown to allow more limited operations, including those utilizing “mini-incisions” under local anesthesia, or patients who are not good risks for general anesthesia.

Improved cosmesis, shorter lengths of stay, and reduced complications are benefits
from this approach. (See Schwartz 10th ed., p. 1564.)

248
Q

Intraoperative, rapid PTH assays provide guidance that all hyper functioning glands have been removed during parathyroidectomy. What criterion is used to indicate satisfactory resolution of the hyperparathyroidism during the procedure?

A. Greater than 50% all in PTH level within 10 minutes of removal of parathyroid tissue.

B. Greater than 25% all in PTH level within 30 minutes of removal of parathyroid tissue.

C. Greater than 75% all in PTH level within 10 minutes of removal of parathyroid tissue.

D. Greater than 90% all in PTH level within 30 minutes of removal of parathyroid tissue.

A

Answer: A

When the PTH level alls by greater than 50% within 10 minutes after removal of parathyroid tissue, the cause of the hyperparathyroidism is likely to have been removed, and the operation can be stopped. (See Schwartz 10th ed., p. 1566.)

249
Q

A patient with persistent ulcer disease is diagnosed with a gastrinoma. Serum chemistry studies indicate hypercalcemia, and an elevated PTH level is documented. What is the indicated course of treatment?

A. Administration of mithramycin 25 mg/kg/day or 4 to 5 days to lower calcium levels.

B. Administration of octreotide 100 mg ID to suppress gastrin secretion.

C. Abdominal exploration for removal of the gastrinoma.

D. Neck exploration for removal of the parathyroid adenoma.

A

Answer: D

In patients with MEN1, hyperparathyroidism should be corrected before treatment of the gastrinoma because resolution of hypercalcemia may allow gastrin levels to all to normal. (See Schwartz 10th ed., p. 1567.)

250
Q

Which of the following findings is suggestive of a parathyroid carcinoma?

A. An elevated serum calcium level greater than 14mg/dL.

B. An elevated PTH level greater than five times normal.

C. A palpable mass in the neck.

D. All of the above.

A

Answer: A

Parathyroid carcinoma occurs in about 1% of patients with primary hyperparathyroidism, or in about 1000 patients per year in the United States.

Findings include an elevated serum calcium level greater than 14 mg/dL, and elevated PTH level greater than five times normal, and a palpable mass in the neck, but none of these may be present.

Complete surgical removal is the most effective therapy with radical neck dissection in lymphadenopathy is present; however, one-third of patients have metastatic disease when first diagnosed.(See Schwartz 10th ed., p. 1570.)

251
Q

A 50-year-old, healthy-appearing man undergoes evaluation of persistent hypertension. Serum chemistries reveal hypokalemia (less than 3.2 mmol/L) and imaging studies reveal a unilateral adrenal mass. What is the likely diagnosis?

A. Secondary hypercortisolism (Cushing disease)

B. Primary hypercortisolism (Cushing syndrome)

C. Hyperaldosteronism (Conn syndrome)

D. Pheochromocytoma

A

Answer: C

Primary aldosteronism, or Conn syndrome, is seen in about 1% o hypertensive patients.

It is more common in middle- aged individual and is usually associated with a single adenoma of the adrenal cortex.

The hypertension is usually refractory to medical treatment, and is classically associated with hypokalemia, but may be seen in normokalemic individuals. (See Schwartz 10th ed., p. 1578.)

252
Q

A 35-year-old woman undergoes an evaluation or infertility. She has gained almost 100 lb in the past year, is hypertensive, and is borderline diabetic. She also complains of easy bruising. Her serum chemistries are normal with the exception of an elevated glucose. Imaging studies reveal a unilateral adrenal mass. What is the likely diagnosis?

A. Secondary hypercortisolism (Cushing disease)

B. Primary hypercortisolism (Cushing syndrome)

C. Hyperaldosteronism (Conn syndrome)

D. Pheochromocytoma

A

Answer: B

Cushing syndrome refers to any cause of hypercortisolism caused by either an adrenal source or exogenous administration of steroids.

Cushing disease refers only to an adrenocorticotropic hormone (ACTH)-secreting adenoma of the pituitary gland.

Cushing syndrome due to an isolated adrenal adenoma is far less common than hypercortisolism due to a pituitary adenoma, but adrenalectomy is curative for primary adrenal tumors or for adrenal hyperplasia that persists despite efforts to resect a pituitary tumor. (See Schwartz 10th ed., p. 1580.)

253
Q

All of the following imaging techniques are useful to localize a pheochromocytoma EXCEPT

A. Computed tomography (CT) scan

B. Magnetic resonance imaging (MRI) scan

C. Metaiodobenzylguanidine (MIBG) scan

D. Octreotide scan

A

Answer: D

Pheochromocytomas are solid tumors which appear on computed tomography (CT) scan as soft tissue masses.

They are detected with 85 to 95% accuracy, but it is important to avoid intravenous-contrast enhancement when a pheochromocytoma is suspected; intravenous contrast can provoke a hyper- tensive crisis due to release of catecholamines. MR is useful to identify pheochromocytomas, both because they identify soft tissue masses, but also because this tumor tends to enhance on 2-weighted images.

Radio-labeled metaiodobenzylguanidine (MIBG) is taken up avidly by the pheochromocytoma because its structure is similar to norepinephrine.

Therefore the MIBG scan can localize an occult tumor.

Octreotide scans are not used or pheochromocytoma as the tumor does not overexpress somatostatin receptors. (See Schwartz 10th ed., p. 1586.)

254
Q

Pheochromocytomas can secrete excess amounts of all of the following EXCEPT

A. Dopa (L dihydroxyphenylalanine)

B. Dopamine

C. Norepinephrine

D. Epinephrine

A

Answer: A

Extra-adrenal pheochromocytomas (also known as paragangliomas) secrete norepinephrine, because these sites lack the enzyme (phenylethanolamine N-methyltrans erase) which converts norepinephrine to epinephrine.

Adrenal pheochromocytomas secrete both epinephrine and norepinephrine as well as dopamine.

Some pheochromocytomas secrete only dopamine, and patients with these tumors may be normotensive. (See Schwartz 10th ed., p. 1586.)

255
Q

The preoperative preparation of a patient with pheochromocytoma should include all of the following EXCEPT

A. An alpha-adrenergic blocker such as phentolamine.

B. A beta-adrenergic blocker such as propranolol.

C. Intravenous hydration to avoid volume depletion.

D. Systemic steroids to avoid adrenal insufficiency.

A

Answer: D

The preoperative preparation of a patient with a catecholamine-secreting tumor includes alpha-hypertension, a beta-adrenergic blocker to prevent tachycardia, and volume replacement to avoid hypotension due to alpha- and beta- blockers.

Steroids are not needed to prevent adrenal insufficiency. (See Schwartz 10th ed., p. 1586.)

256
Q

A follow-up CT scan in a 60-year-old patient with previous nephrolithiasis reveals a 1.5-cm hypovascular round lesion with clear margins in the right adrenal gland. The patient is not hypertensive, hyperglycemic, or hypokalemic. Urinary catechol metabolites are within normal limits, and serum cortisol and ACTH levels are normal. Which course is advisable?

A. Repeat CT scan and chemical tests annually.

B. Percutaneous FNAB.

C. Adrenal venous sampling or cortisol, renin, and
angiotensin.

D. Laparoscopic adrenalectomy.

A

Answer: A

The adrenal “incidentaloma” is an increasingly common finding with the ubiquitous use of CT scanning, with an incidence of 0.4 to 4.4%.

A variety of benign and malignant lesions can account for these findings, and a distant history of malignancy elsewhere should raise the possibility of metastatic disease.

Primary malignancy of the adrenal gland is rare, and the functioning tumors are excluded by screening tests or cortisol and catecholamine excess.

In the absence of symptoms associated with adrenal disease, annual follow-up of these lesions with imaging and chemical tests seems prudent. (See Schwartz 10th ed., p. 1589.)

257
Q

Advantages of laparoscopic adrenalectomy compared with open adrenalectomy include all of the following EXCEPT

A. Decreased incidence of wound infection

B. Decreased length of hospital stay

C. Decreased operative time

D. Decreased narcotic analgesic use

A

Answer: C

Laparoscopic (videoscopic) approaches to adrenalectomy have been shown to be advantageous or several outcomes including wound complications, analgesic use, and length of hospital stay.

These advantages are in balance to adverse considerations including length of operative time and cost. (See Schwartz 10th ed., p. 1591.)

258
Q

In patients who undergo bilateral adrenalectomy in treat- ment of Cushing disease after failed attempts at resection of an ACTH-secreting pituitary adenoma, the subsequent development of Nelson syndrome is associated with all of the following EXCEPT

A. Hyperpigmentation
B. Diminished visual fields
C. Loss of hearing
D. Headaches

A

Answer: C

Nelson syndrome describes symptoms due to the progressive enlargement of a persistent ACTH-secreting pituitary fossa tumor.

These symptoms include hyperpigmentation, visual
field loss, headaches, and extraocular muscle palsies.

Interference with the olfactory nerve is not part of the syndrome. (See Schwartz 10th ed., p. 1594.)

259
Q

When performing a thyroidectomy, which of the following anatomic considerations is incorrect:

A. The middle thyroid veins drain into the internal jugular vein.

B. The inferior thyroid artery arises directly from the external carotid artery.

C. The thyroidea ima artery arises directly from the aorta in 1%–4% of patients.

D. The ligament of Berry is located near the entry point of the recurrent laryngeal nerve (RLN).

E. Venous drainage of the thyroid gland is via the superior, middle, and inferior branches.

A

ANSWER: B

COMMENTS: The arterial blood supply of the thyroid gland is provided by two main arterial branches.

The superior thyroid artery arises from the external carotid artery and gives off multiple branches that are collectively referred to as the superior pole vessels.

The inferior thyroid artery is given off as a branch of the thyrocervical trunk, which originates from the subclavian artery.

The ligament of Berry is a condensation of the thyroid capsule located on the posterior surface of the gland.

It is a tough band of tissue that passes either anterior or posterior to the course of the RLN as it travels into the cricothyroid membrane.

The superior, middle, and inferior thyroid veins drain the thyroid gland.

The superior and middle thyroid veins eventually drain into the internal jugular vein, while the inferior thyroid veins drain into the innominate and brachiocephalic veins.

The distal end of the thyroglossal duct persists in approximately 50% of people.

This remnant is anatomically referred to as the pyramidal lobe.

260
Q

A 45-year-old woman has chronic neck pain and a newly diagnosed posterior pharyngeal neck mass found on magnetic resonance imaging (MRI) (see image of transoral examination— Fig. 16.1). The next best step in the appropriate management of this patient is:

A. Computed tomography (CT)

B. Cervical ultrasound

C. Physical examination with observed swallowing

D. Radioiodine uptake scan

E. Observation

A

ANSWER: C
COMMENTS: The most likely diagnosis in this patient is a lingual thyroid. This abnormality arises when the thyroid fails to descend normally into the standard cervical position. In some patients, this may be the only thyroid tissue present. It originates from the base of the tongue in the region of the foramen cecum. The diagnosis is best made by a physical examination that demon- strates a midline anterior cervical mass that moves with protrusion of the tongue as opposed to deglutition. Cervical ultrasound dem- onstrating an absence of thyroid tissue is also highly suggestive of a lingual thyroid. A radioactive uptake scan demonstrates the uptake of iodine within the mass. If the gland is enlarged, patients may have obstructive symptoms such as dysphagia, choking, or airway obstruction. Management of these patients is typically excision.

261
Q

A 46-year-old male is undergoing an anterior cervical disk fusion. Which of the following is correct with regard to the RLN?

A. The left RLN loops around the subclavian artery and ascends medially into the neck.

B. The right RLN loops around the inferior thyroid artery, ascends laterally to medially, and enters the cricothyroid membrane.

C. The right inferior laryngeal nerve is nonrecurrent in 0.5%–1% of patients.

D. The RLNs innervate the true vocal cords and the cricothyroid muscles.

E. The medial branch of the RLN is primarily sensory.

A

ANSWER: C
COMMENTS: The RLN arises from the vagus nerve after passing into the mediastinum. The left RLN encircles the aortic arch at the ductus arteriosus and then ascends medially in the neck within the tracheoesophageal groove. The right RLN encir- cles the right subclavian artery and ascends laterally to medially toward the tracheoesophageal groove. Once in the central aspect of the neck, the RLN nerve can branch into medial and lateral components. The medial branch typically carries the motor fibers and is at risk for injury along its course toward the crico- thyroid membrane if not identified as a separate structure rela- tive to the lateral sensory branch. The laryngeal nerves can be nonrecurring in approximately 0.5%–1% of patients. In this case, the nerve originates from the cervical position of the vagus nerve. The risk for injury during cervical procedures is higher when the nerve is nonrecurrent because it travels perpendicular rather than parallel to the tracheoesophageal groove. It is more common on the right and is most often associated with the presence of aberrant cervical vascular anatomy such as a retroesophageal subclavian artery. On the left, it is rarer as it is associated with situs inversus and a right-sided aortic arch. The RLN innervates all of the intrinsic muscles of the larynx except the cricothyroid muscle, which is innervated by the external branch of the superior laryngeal nerve.

262
Q

With regard to thyroid hormone synthesis and uptake, which
of the following is correct?

A. Iodine trapping involves endocytosis of circulating iodine particles.

B. In the euthyroid state, Triiodothyronine (T3) is the main hormone produced by the thyroid.

C. Thyroid peroxidase is responsible for the peripheral conversion of T4 to thyroxine (T3).

D. Thyroglobulin is a glycoprotein synthesized in the rough endoplasmic reticulum of the thyrocyte.
E. The primary site of peripheral deiodination of T4 to the active form T3 occurs in the adrenal gland.

A

ANSWER: D
COMMENTS: The synthesis and uptake of thyroid hormones con- sists of several steps. It begins with iodine trapping via adenosine triphosphate–dependent transport across the basement membrane of the thyrocyte. Thyroglobulin, a glycoprotein synthesized in the rough endoplasmic reticulum, then becomes iodinated by an intracellularly located catalytic enzyme called thyroid peroxidase. Two diiodotyro- sines are coupled to form T4. Peripherally, T4 is converted to the active thyroid hormone T3. In the euthyroid state, T4 is the predominant hormone produced by the thyroid gland. Peripheral deiodination of T4 to the active form T3 occurs in the liver, muscle, kidney, and anterior pituitary. T3 is transported in the serum bound to circulating carrier proteins such as thyroxine-binding globulin (TBG) and albumin.

263
Q

Hypothyroidism can be associated with all of the following pharmacologic therapies except:

A. Lithium

B. Amiodarone

C. Interleukin-2

D. Propylthiouracil (PTU)

E. Cimetidine

A

ANSWER: E

COMMENTS: Hypothyroidism has been associated with a number of pharmacologic therapies, the most frequent being lithium. The mechanism involves blockage of the cyclic adenosine monophos- phate–dependent pathway of hormone synthesis. It is also seen with other agents, including amiodarone, cytokines, and antithyroid medications. No relationship has been demonstrated between hypo- thyroidism and cimetidine.

264
Q

With regard to the pharmacologic treatment of hyperthyroid-
ism, which of the following is incorrect?

A. PTU works by inhibiting organic binding of iodine and coupling of iodotyrosines.

B. PTU is associated with agranulocytosis.

C. PTU is the preferred treatment in pregnant patients.

D. Methimazole can worsen exophthalmos in patients with Graves’ disease.

E. Methimazole has a longer half-life and only requires once-daily dosing.

A

ANSWER: D

COMMENTS: The two most commonly prescribed antithyroid medications in the treatment of Graves’ disease are methimazole and PTU. Both agents work by blocking organic binding of iodine and coupling of iodotyrosines. PTU also inhibits peripheral conver- sion of T4 to T3. Both have a side-effect profile that includes hepa- totoxicity and reversible granulocytopenia. Methimazole is favored since it requires once-daily dosing as a result of its longer half-life. Both cross the placenta during pregnancy, but PTU appears to exhibit less transplacental transfer and has a lower toxicity profile. Radioactive iodine therapy, an alternative to surgery or medical therapy, can worsen exophthalmos following treatment, but neither antithyroid medication alters the degree of the eye findings associ- ated with Graves’ disease.

265
Q

All of the following are extrathyroidal manifestations of
Graves’ disease except:

A. Vitiligo

B. Pretibial myxedema

C. Exophthalmos

D. Myxedema coma

E. Acropachy

A

ANSWER: D

COMMENTS: Graves’ disease is an example of an organ-specific autoimmune disease. It has been associated with other autoimmune diseases such as pernicious anemia, vitiligo, alopecia, angioedema, and myasthenia gravis. The extrathyroidal manifestations of Graves’ disease include exophthalmos, pretibial myxedema, acro- pachy, and vitiligo. Patients with severe hypothyroidism may present with myxedema coma and can require initial emergency treatment with high doses of IV T4.

266
Q

Which of the following antibodies is diagnostic of patients
with Graves’ disease?

A. Thyroglobulin antibodies (anti-Tg)

B. Thyroid peroxidase antibodies (anti-TPO)

C. Anticardiolipin antibodies

D. Thyroid-stimulating antibodies (anti-TSH)

E. Antimicrosomal antibodies

A

ANSWER: D

COMMENTS: A common feature of autoimmune thyroid disease is the presence of immunoreactivity toward specific thyroid anti- gens. More than 90% of patients with Graves’ disease will express elevated thyroid-stimulating antibodies (anti-TSH), which is considered diagnostic. Thyroid antibodies also include thyroid peroxidase antibodies (anti-TPO) and thyroglobulin antibodies (anti-Tg), which typically indicate an autoimmune thyroiditis, but not thyroid function. Rarely in Graves’ disease do patients have antinuclear antibodies such as anti-Ro, anti-dsDNA, or anticar- diolipin antibodies

267
Q

A 42-year-old woman complains to her physician of symptoms associated with hyperthyroidism. On examination she has a palpable nodule, but no evidence of exophthalmos. She does have pretibial myxedema. Her laboratory workup reveals a suppressed thyroid stimulating hormone (TSH) level with elevated free T3. What is the next best step in the management of this patient?

A. Radioactive 123I uptake scan
B. Thyroid peroxidase antibodies (anti-TPO) C. PTU
D. Fine-needle aspiration (FNA)
E. Cervical ultrasound

A

ANSWER: A

268
Q

Which of the following is not an acceptable indication for surgical treatment of hyperthyroidism of the patient in Question 9?

A. A nodule confirmed or suspicious for malignancy

B. Pretibial myxedema

C. Noncompliance with medical management

D. Age younger than 15 years

E. Severe Graves’ ophthalmopathy

A

ANSWER: B

COMMENTS: A diagnosis of thyrotoxicosis is suspected in this patient. Of the tests listed, the one most likely to yield a diagnosis is a radioactive uptake scan. The initial workup of a thyroid nodule should not include a thyroid uptake scan unless the patient exhibits signs or symptoms of hyperthyroidism. Elevated uptake with a diffusely enlarged gland confirms the diagnosis of Graves’ disease and assists in differentiating it from other forms of hyperthyroidism. In this patient, the palp- able mass on examination could instead be a toxic nodule, and the radioactive uptake scan would illustrate a “hot” nodule in the setting of a suppressed gland. A thyroid uptake scan is useful in determining whether the patient has a toxic nodular goiter as a potential explanation of her hyperthyroid state. Cervi- cal ultrasound depicts thyroid anatomy, but does not indicate thyroid function. The most useful test in the initial screening of patients for hyperthyroidism is TSH since the suppression of TSH below normal is a sensitive indicator of excess circulating thyroid hormone. It would not be appropriate to initiate therapy for hyperthyroidism, however, until the cause of the hyperthy- roidism is established. Thyroid peroxidase antibodies (anti- TPO) are commonly found in Graves’ disease, but are not diagnostic. A biopsy by FNA would be appropriate if the nodules were found to have no uptake on a radioiodine uptake scan. Treatment of hyperthyroidism includes medical therapy, radioactive iodine, and surgical resection. Presently, the major- ity of patients with thyrotoxicosis are treated with pharmaco- logic agents or, alternatively, ablation with radioactive iodine. Surgical treatment is recommended for patients who have (1) a large goiter with a low radioiodine uptake, (2) noncompliance with medical management, (3) suspicion of malignancy, (4) severe exophthalmos, (5) pregnancy, and (6) age younger than 15 years (because of the risks associated with exposure to radiation).

269
Q

With regard to Hashimoto’s thyroiditis, which of the
following is correct?

A. The majority of patients are transiently hypothyroid but with time return to a euthyroid state.

B. It is primarily treated surgically.

C. Radioactive iodine is useful in the treatment of Hashi- moto’s thyroiditis.

D. Thyroid microsomal antibodies are detected in the serum of patients.

E. Hashimoto’s thyroiditis is more common in men than in women.

A

ANSWER: D

COMMENTS: Hashimoto’s disease is a type of autoimmune thyroiditis. Histologically, the gland exhibits a dense lymphocytic infiltrate associated with a dense fibrosis. When the disease is active, patients can experience symptoms associated with either hypothyroidism or hyperthyroidism. The natural progression of the disease is the eventual development of a multinodular goiter associ- ated with hypothyroidism. Surgery is rarely indicated except in the setting of (1) debilitating symptoms of thyroid dysfunction, (2) compressive symptoms from multinodular goiter, and (3) suspicion of malignancy. The presence of a nodular disease mandates FNA to rule out lymphoma or papillary thyroid carcinoma. Hashimoto’s thyroiditis is more common in women than in men (4 to 10:1 female preponderance).

270
Q

All of the following statements regarding Hürthle cell
carcinoma are correct except:

A. It represents a subtype of papillary thyroid carcinoma.

B. It is associated with a higher mortality rate.

C. It is more likely to be multifocal compared with follicular carcinoma

D. It demonstrates poor radioactive iodine uptake.

E. It is more likely than follicular carcinoma to have lymph
node metastases.

A

ANSWER: A

COMMENTS: Hürthle cell carcinoma is considered a subtype of follicular carcinoma and, similarly, is characterized by vascular or capsular invasion. Hürthle cell carcinomas account for approxi- mately 3% of all thyroid malignancies. Different from follicular carcinoma, Hürthle cell carcinomas are more often multifocal and bilateral, have a higher rate of local nodal metastases (25%), and demonstrate a poor radioactive iodine uptake. Hürthle cell carcino- mas have also been associated with higher mortality than follicular carcinomas (approximately 20% at 10 years). Previous radiation exposure has been correlated with an increase in bilateralism and multicentricity of Hürthle cell neoplasms, as well as an increased incidence of contralateral non–Hürthle cell malignant thyroid lesions.

271
Q

A patient is undergoing a planned total thyroidectomy for bilateral thyroid nodules, of which the right nodule was consistent with a follicular neoplasm with Hürthle cell features on FNA. During initial mobilization of the gland on the right side, the right RLN was unintentionally transected. What is the best next step in the management of this patient?

A. Repair RLN primarily.

B. Perform a frozen section of the contralateral nodule and proceed with total thyroidectomy only if the biopsy specimen suggests malignancy.

C. Perform right lobectomy and isthmusectomy.

D. Perform left subtotal lobectomy.

E. Perform right lobectomy with nodulectomy of the lesion located on the left.

A

ANSWER: C

COMMENTS: Injury to the RLN occurs in less than 1% of patients. The risk is higher with low-volume surgeons, reoperative surgery, irradiated necks, and invasive cancers. Transient neura- praxia can occur as a result of excessive stretch on the nerve, but typically recovers following the procedure. In such cases, there can be a loss of signal with intraoperative nerve monitoring, just as would be seen if the nerves were completely transected. One should repair the nerve primarily or using a nerve graft if necessary, typi- cally from ansa cervicalis, in order to decrease tension on the repair at the same operation. The goal of repairing the nerve is to maintain tone to the false vocal cords, thereby preventing it from atrophying over time. This, however, should be completed after the ipsilateral lobe of the thyroid is removed to prevent the repair from being disturbed. If this problem is encountered in a patient such as this one in whom the intended operation is not yet complete, the best decision is to complete the procedure on the side where the nerve has been injured and not to proceed to the unaffected side since a diagnosis of malignancy has not yet been established. After the recovery period, the patient should undergo a formal vocal cord evaluation and have a lengthy discussion regarding the risks and benefits of proceeding with the subsequent portion of the operation if necessary. Intraoperative laryngoscopy is of no use because the patient is unresponsive, and a functional vocal cord evaluation is very limited while the patient is still intubated.
`

272
Q

With regard to the pathologic features of thyroid carcinoma,
which of the following is correct?

A. Psammoma bodies are a feature of medullary thyroid carcinoma (MTC).

B. Hürthle cell carcinoma represents a subtype of anaplastic thyroid carcinoma.

C. Amyloid deposits are a characteristic of papillary thyroid carcinoma.

D. MTC typically spreads hematogenously.

E. Nuclear grooves and inclusions are a characteristic
feature of papillary thyroid carcinoma.

A

ANSWER: E

COMMENTS: The pathologic characteristics of papillary thyroid carcinoma include nuclear grooves and inclusions. Psammoma bodies are also pathognomonic of papillary thyroid carcinoma. Papillary thyroid carcinoma spreads via the lymphatics, while fol- licular thyroid carcinoma spreads hematogenously. MTC spreads via the lymphatics initially. Amyloid deposits represent collections of calcitonin within the thyroid specimen. Hürthle cell carcinoma is a subtype of follicular thyroid carcinoma that has a more aggres- sive phenotype.

273
Q

A 72-year-old woman with Hashimoto’s thyroiditis is evaluated for a rapidly enlarging neck mass. The patient takes levothyroxine replacement. Despite no change in her medication dosage, she has been experiencing fevers, night sweats, and weight loss. Ultrasound reveals a 4-cm left thyroid mass with a pseudocystic pattern. FNA is nondiag- nostic. What is the next step in the management of this patient?

A. Nonsteroidal antiinflammatory drugs

B. Repeated FNA

C. Radioactive iodine

D. Open or core biopsy

E. Increased dose of levothyroxine with follow-up ultra- sound in 6 months

A

ANSWER: D

COMMENTS: Primary thyroid lymphoma is rare (<1% of all thyroid malignancies). There is an increased association between lymphoma and Hashimoto’s thyroiditis. The initial workup of a newly diagnosed thyroid mass should be completed. FNA can be diagnostic, but flow cytometry is needed to establish the diagnosis. On ultrasound, the thyroid will classically have a pseudocystic pattern. In the current patient, the presence of Hashimoto’s thyroid- itis, a rapidly enlarging mass, and the constitutional symptoms are highly suggestive of primary thyroid lymphoma. If FNA fails to yield a diagnosis on the first attempt, it has been shown that repeated FNA is unlikely to be diagnostic in subsequent attempts. Observation is not an appropriate step in the management of any patient with a rapidly enlarging neck mass. Performing open or core biopsy is a feasible option in these circumstances and would more likely provide an adequate cell yield to complete flow cytometry and establish monoclonality. Treatment varies between chemother- apy and surgical ablation. Resection is typically reserved for patients who fail to respond to chemotherapy or have completed their course of adjuvant therapy and demonstrated incomplete regression of disease. If surgery is planned, total thyroidectomy should be performed, followed by chemotherapy.

274
Q

Which of the following genes has been associated with a less favorable prognosis in patients with papillary thyroid carcinoma?

A. RET protooncogene

B. Ras

C. BRAF

D. Menin

E. p53

A

ANSWER: C

COMMENTS: Papillary thyroid carcinoma is the most common type of thyroid malignancy. Patients have an overall predicted 5-year survival rate greater than 95%. Several clinical factors are currently used as prognosticators of a patient’s clinical outcome. B-type RAF (BRAF) is part of the Raf kinase family and plays a fundamental role in the classic intracellular signaling mitogen- activated protein kinase (MAPK) pathway. BRAF mutations occur in papillary and anaplastic tumors, but not in follicular thyroid carcinomas. BRAF mutations are associated with an increased risk for LN metastasis, extrathyroidal invasion, and advanced tumor stages. RET is the gene found to be mutated in patients with MTC. RET can also fuse with other genes, and such fusion products have been implicated in the pathogenesis of papillary thyroid carcinoma. Menin is mutated in patients with multiple endocrine neoplasia type I. Both p53 and RAS have not been identified as genetic prognos- ticators in patients with papillary thyroid carcinoma.

275
Q

An 82-year-old female with a long-standing history of a neck mass presents because of sudden rapid painful neck enlarge- ment associated with a change in her voice and dysphagia. FNA biopsy reveals giant and multinucleated cells. All of the following are appropriate steps in the management of this patient except:

A. Immediate tracheostomy

B. Total thyroidectomy with LN dissection for intrathyroidal tumors

C. Cytotoxic chemotherapy

D. Adjuvant radiation if performance status permits

E. En bloc resection for tumors with extrathyroidal extension

A

ANSWER: A
COMMENTS: Approximately 1% of thyroid malignancies in the United States are discovered to be anaplastic thyroid carcinoma. Elderly women are more commonly affected. The tumors are often large and may be fixed to surrounding structures with areas of necrosis that can be visualized on MRI. Diagnosis is confirmed with the above FNA biopsy result. All patients should undergo a preoperative laryngoscopy to assess the status of the vocal cords. A total thyroidectomy with LN dissection is recommended for intrathyroidal masses. En bloc resection should be considered if an R1 resection can be achieved for tumors with extrathyroidal exten- sion. Cytotoxic chemotherapy is typically given along with adju- vant radiation for patients with a good performance status and no evidence of metastatic disease. Tracheostomy should be avoided for as long as possible unless there is impending airway loss.

276
Q

With regard to thyroid metastases, which of the following malignancies most commonly spreads to the thyroid?

A. Renal cell carcinoma

B. Breast cancer

C. Colon cancer

D. Lung cancer

E. Melanoma

A

ANSWER: A

COMMENTS: Metastases to the thyroid are rare. The most common metastases to the thyroid are from renal cell carcinoma, and they can develop several years after the initial diagnosis. Other types of cancer that metastasize to the thyroid include lung and breast cancers. Management depends on the state of the primary disease in other locations and the predicted overall survival.

277
Q

All of the following are considered an increased risk factor for cancer in a patient with a thyroid mass except:

A. Age younger than 45 years

B. Rapid growth

C. Family history

D. Hot nodules on thyroid uptake scan

E. Male gender

A

ANSWER: D
COMMENTS: There is an associated increased risk for thyroid malignancy in patients with thyroid nodules and any of the follow- ing features: radiation exposure, rapid growth during observation, family history of thyroid cancer, cold nodule on radioactive uptake scan, male gender, and age older than 45 years. Hot nodules on a thyroid uptake scan generally indicate hyperfunctioning growth associated with such conditions as toxic multinodular goiter or Graves’ disease.

278
Q

A 57-year-old female presents with a new diagnosis of a thyroid nodule. Routine workup includes all of the following except:

A. Physical examination

B. TSH

C. Cervical ultrasound

D. Thyroglobulin

E. Total thyroxine (T4)

A

ANSWER: D
COMMENTS: The initial workup of a new thyroid nodule involves obtaining a thorough history and physical examination, with particular attention to the sequelae of hyper- and hypothyroid- ism, symptoms of compression, and signs of malignancy. The physical examination focuses on palpation of the thyroid and cervi- cal lymphadenopathy. Ultrasonography should be performed to document whether a nodule is suspicious and warrants further workup before surgery. If a diagnosis of malignancy is made, the extent of surgery (subtotal versus complete versus LN dissection) can be determined preoperatively. Thyroid dysfunction involves measurement of TSH. The thyroid production of T4 is assessed by measuring both free and total T4. Total T4 is the amount of both free and protein-bound hormone. Thyroglobulin is not routinely measured as part of the initial assessment of thyroid function. Obtaining a family history of thyroid dysfunction or malignancy and a history of radiation are other key aspects of the history taking and physical examination.

279
Q

Calcitonin is produced by the parafollicular cells of the thyroid gland. Measurement of calcitonin is most useful in what disease process?

A. Pheochromocytoma

B. Follicular thyroid carcinoma

C. Hashimoto’s disease

D. MTC

E. Papillary thyroid carcinoma

A

ANSWER: D
COMMENTS: Calcitonin is produced by the parafollicular cells of the thyroid gland. This 32–amino acid polypeptide is the princi- pal hormone responsible for lowering serum calcium levels during states of hypercalcemia. The action of calcitonin takes place on the surface receptors of osteoclasts, which work to inhibit bone resorp- tion and thus reduce serum calcium levels. MTCs are tumors of the parafollicular cells that produce calcitonin. The resulting hyper- calcitoninemia seen in these patients can be a measure of tumor burden. Carcinoembryonic antigen (CEA) may also be elevated in some patients with MTC. Calcitonin doubling times of greater than 2 years are associated with a better long-term prognosis than the doubling times of less than 6 months. The American Thyroid Asso- ciation (ATA) provides a web-based calculator to monitor the levels of calcitonin and CEA to aid in the follow-up and management of patients with MTC.

280
Q

In the management of thyrotoxicosis, which of the following is correct?

A. Iodine given in large doses stimulates the release of thyroid hormone.

B. A euthyroid state should be achieved through the use of antithyroid drugs before surgery.

C. Corticosteroids stimulate the peripheral conversion of T4 to T3.

D. β-Blockers potentiate the effects of thyroid hormone through adrenergic stimulation of thyroid receptors.

E. Supersaturated potassium iodide (SSKI) should not be administered preoperatively.

A

ANSWER: B
COMMENTS: Preoperatively, patients with Graves’ disease should have their thyrotoxic state controlled through the use of antithyroid medications. Without this, induction of anesthesia can induce a thyroid storm. Many endocrine surgeons advocate giving patients a 7- to 10-day preoperative course of either Lugol’s solution or SSKI to help diminish the hypervascularity of the gland and minimize operative blood loss, although some studies have shown that preoperative administration of concentrated iodide preparations can also induce hypertrophy of the gland and should possibly be avoided in patients with diffusely enlarged toxic goiters. When given in concentrated doses, iodine can inhibit the release of thyroid hormone. This process of “thyroid stunning” is referred to as the Wolff–Chaikoff effect. It is sometimes seen in patients who have a diagnostic radioactive iodine scan and then demonstrate minimal uptake of radioactive iodine when the thera- peutic dose is given. In preparing hyperthyroid patients for surgery, administration of potassium iodide solutions has been shown to treat hyperactivity and decrease intraoperative blood loss during thyroidectomy. This stunning effect is transient and is thus not used for the long-term management of hyperthyroid patients. Concentrated potassium iodide (Lugol’s solution) should be used with caution in this setting since it will aggravate the hyperthyroidism after its initial effect wears off. β-Blockers are used to mitigate the symptoms of adrenergic stimulation but are not given to all patients with Graves’ disease preoperatively unless they exhibit signs of thyrotoxic-induced tachycardia. β-Blockers do not directly inhibit synthesis of thyroid hormone, but they do counteract some of the peripheral effects of its action on the car- diovascular system. Corticosteroids not only inhibit peripheral conversion of T4 to T3 but also suppress the production of TSH. In acute severe hyperthyroid states, corticosteroids can help in the initial treatment.

281
Q

A 25-year-old woman at 10 weeks’ gestation has increasing shortness of breath and anxiety. The clinician wishes to screen her for hyperthyroidism. Which of the following statements is relevant to the interpretation of thyroid function in pregnant patients?

A. TBG is decreased and thus levels of total T4 and T3 are increased.

B. Decreased renal iodine clearance causes a reciprocal decrease in total T3.

C. Increased plasma volume decreases the total T4 and T3 levels measured in serum.

D. A first-trimester increase in human chorionic gonadotro- pin (hCG) causes a reciprocal decrease in TSH levels.

E. Thyrotoxicosis is relatively common in the first and second trimesters of pregnancy.

A

ANSWER: D

COMMENTS: Thyrotoxicosis occurs in approximately 0.1%– 0.4% of pregnancies in the United States. If not recognized and treated promptly, severe complications can arise in both the mother and the fetus. The hormonal changes and fluctuating metabolic demands during pregnancy have an important impact on thyroid physiology. These changes can make interpretation of laboratory tests of thyroid function in pregnant patients extremely challeng- ing. During pregnancy, there is a rise in TBG, which in turn causes an increase in the total serum T4 and T3. The degree of change in TBG levels is dependent on the particular trimester. By 16 to 20 weeks, serum TBG levels have in most cases risen to double the nonpregnant values. Additionally, hCG shares structural similarity with TSH and has been shown to have weak thyroid-stimulating activity. This fact may explain why thyroid nodules often increase in size during pregnancy. The mimicked activity of TSH allows hCG to provide negative feedback in such a way that TSH levels during pregnancy are always lower than is reflective of the true thyroid functional state. Thus if normal ranges of TSH that are specific to pregnancy are not used, hyperthyroidism can be inap- propriately diagnosed. Additionally, in pregnancy, renal clearance of iodine is increased, and thus patients will have increased base- line 24-h radioactive iodine uptake when compared with nonpreg- nant patients. The increased renal clearance has no direct effect on the circulating levels of total T3. In fact, pregnant patients have increased T4 and T3 levels because of their increased total plasma volumes.

282
Q

A 37-year-old woman has an incidentally discovered thyroid mass on CT performed during a recent visit to the emergency department for workup of a fall at work. The patient undergoes neck ultrasonography. Which of the following features is less likely to be a malignant lesion?

A. Incomplete halo

B. Peripheral calcifications

C. Hypoechoic lesion

D. Irregular margins

E. Size smaller than 2 cm

A

ANSWER: B
COMMENTS: In a clinically euthyroid patient, the initial test of choice should be cervical ultrasound followed by FNA of any suspicious nodules discovered on clinical or radiographic examina- tion. FNA has a false-negative rate of approximately 3% and a false-positive rate of 4%. A TSH level should be ordered to docu- ment thyroid function. Not all calcifications on thyroid ultrasound are associated with a malignant lesion. Typical benign cystic lesions will exhibit a complete halo (intact capsule) and peripheral calcifi- cations. Size is not generally thought to be a consistent predictor of malignancy. However, lesions greater than 1 cm are believed to be amenable to fine-needle biopsy. Table 16.1 lists some of the ultrasonographic features associated with benign and malignant disease.

283
Q

The patient in Question 16 is found to have a 2-cm dominant nodule located in the right thyroid lobe. Cytologic evaluation of an FNA biopsy specimen reveals “atypia of undetermined significance.” All of the following are acceptable next steps in her management except:

A. Right thyroid lobectomy

B. Observation with repeat ultrasound and FNA in 6 months

C. Right thyroid lobectomy with an intraoperative frozen section

D. Radioactive iodine ablation

E. Gene expression analysis of thyroid nodule FNA aspirate

A

ANSWER: D
COMMENTS: A finding of follicular neoplasm on FNA carries approximately 15%–30% risk of harboring a malignancy. At a minimum, the patient should undergo a right lobectomy with or without a frozen resection. Surgical excision is necessary because follicular neoplasms cannot be diagnosed as malignant or benign without microscopic examination of the nodule’s capsule. In patients who have additional risk factors for thyroid cancer (e.g., family history and exposure to radiation), total thyroidectomy should strongly be considered to avoid the need for reoperative surgery given the higher likelihood of finding a malignant nodule. Only patients with atypia of undetermined significance should be offered the option of observation and repeated imaging in 6 months. Expression arrays, which are being used to investigate microRNAs that have been impli- cated for their role in carcinogenesis, are sometimes used to aid in the determination of the risk of malignancy for inconclusive FNA biopsies although their utility has yet to be definitely established.

284
Q

Overall, papillary thyroid carcinoma carries a favorable prognosis. Which of the following is not a prognostic risk factor in the tumor/node/metastasis (TNM) staging system?

A. LN metastases

B. Synchronous bone metastases

C. Serum thyroglobulin level

D. Extranodal extension

E. Vascular invasion

A

ANSWER: C

COMMENTS: Papillary thyroid carcinoma carries a very favor- able prognosis with a 10-year survival of greater than 95%. Risk stratification is a fundamental aspect of how treatment and surveil- lance algorithms are developed. For well-differentiated thyroid carcinomas, the TNM staging is divided into those younger than 45 years old and those 45 years or older. For those younger than 45 years, stage I includes any T or N and M0 and stage II includes any T or N and M1. For those 45 years or older, stage I includes T1, N0, and M0; stage II includes T2 or T3, N0, and M0; stage III includes T4, N0, M0, and any T, N1, M0; and stage IV includes any T or N, and M1. All undifferentiated or anaplastic carcinomas are stage IV. In addition to the TNM staging system, other fre- quently used systems include AGES (age, grade, extrathyroidal extension, and size) and AMES (age, metastases, extension, and size). Although each of these staging systems does a reasonable job of predicting cancer-specific survival, none have any prognostic value in assessing the risk of recurrence. Thyroglobulin levels are not a part of any of the current staging systems and are most useful in the surveillance for recurrence of thyroid cancer after thyroidec- tomy and/or radioactive iodine therapy.

285
Q

A 35-year-old man is newly diagnosed with MTC and is found to have an elevated calcitonin. In regard to the management of this patient, all are correct except:

A. Preoperative ultrasound for LN mapping of the lateral and central compartments should be performed routinely.

B. Biochemical screening for associated endocrinopathies (e.g., hyperparathyroidism or pheochromocytoma) is part of the preoperative workup.

C. Treatment consists of total thyroidectomy with bilateral central node dissection.

D. Measurement of CEA is not a useful marker of disease burden.

E. Genetic counseling and screening should be offered to the patient and his immediate family members.

A

ANSWER: D
COMMENTS: MTC arises from the parafollicular cells of the thyroid. Its serum tumor marker is calcitonin. Patients with diffuse and advanced disease can have symptoms of calcitonin excess (diarrhea, flushing, and abdominal cramping). More than 80% of cases of MTC are sporadic and thus have no hereditary association. That being said, every patient in whom MTC is newly diagnosed should be referred for genetic counseling and evaluation for a RET protooncogene mutation because there are identifiable genetic mutations that predict the prognosis and aggressiveness of the disease. The overall survival of patients with MTC is determined largely by the completeness of resection and the presence of per- sistent disease. Thus surgical treatment of MTC tends to favor aggressive therapy. All patients with biopsy-confirmed MTC and no evidence of nodal or distant metastases should undergo a minimum total thyroidectomy with bilateral central node dis- section. Whether lateral compartment dissection is performed is dependent on whether there is radiographic or operative evidence of gross disease. Current 2015 ATA guidelines for MTC recom- mend that all patients with findings on FNA or a calcitonin level suggestive of MTC should undergo preoperative neck ultrasonog- raphy for the evaluation of LN metastases. Biochemical screening for associated manifestations (e.g., hyperparathyroidism and pheo- chromocytoma) should also be performed by obtaining a serum calcium level and measurement of either plasma metanephrines or 24-h urine catecholamines. CEA and calcitonin are useful tumor markers in predicting disease burden and recurrence in patients with MTC.

286
Q

Recent studies have shown that the incidence of thyroid cancer has nearly doubled in the last two decades. Plausible explanations given by epidemiologists include all of the following except:

A. Childhood radiation exposure

B. Iodine deficiency

C. Estrogen hormone

D. Increased use of diagnostic imaging modalities

E. Increased use of thyroid function tests

A

ANSWER: E
COMMENTS: According to the 2016 Cancer Facts and Figures published by the American Cancer Society, the incidence of thyroid cancer has not only been rising worldwide over the past few decades but is now the most rapidly increasing cancer in the United States. In the United States, rates have increased by over 5% per year from 2003 to 2012. Current evidence supports the fact that the increase is most likely multifactorial, including radiation exposure, environmental, hormonal, and genetic factors. The rise is also partly attributed to increased detection as a result of both an increased use of imaging and more sensitive diagnostic modalities. The iodine status does appear to influence the subtype of thyroid cancer that populations are at risk for. There is a reported increased risk for thyroid cancer in iodine-replete areas, mainly papillary thyroid carcinoma, whereas iodine-deficient areas show a slightly increased incidence of follicular thyroid carcinoma. The utilization of thyroid function tests is not directly associated with the rise in the incidence of thyroid cancer.

287
Q

With regard to parathyroid anatomy, which of the following is incorrect?

A. The superior parathyroid glands are lateral and posterior to the recurrent laryngeal nerve (RLN).

B. Parathyroid glands derive the majority of their blood supply from branches of the superior thyroid artery.

C. The inferior parathyroid glands are medial and anterior to the RLN.

D. Parathyroid glands are drained by the superior, middle, and inferior thyroid veins.

E. Branches from the thyroidea ima may provide arterial blood supply to the parathyroid glands.

A

ANSWER: B
COMMENTS: There are typically four parathyroid glands. Super- numerary glands have been found in up to 13% of people on autopsy. They are typically light brown in appearance, measure 7 mm in size, and each weighs 50 mg. The superior glands are dorsal and lateral to the RLN, while the inferior glands are ventral and medial to the RLN. Parathyroid glands primarily derive their blood supply from the inferior thyroid artery. Branches of the superior thyroid artery can provide up to 20% of the blood supply of the superior glands. Branches from the thyroidea ima, in addition to vessels to the trachea, esophagus, larynx, and mediastinum, may also be found.

288
Q

With regard to calcium homeostasis, which of the following statements is incorrect?

A. Calcium is the most abundant cation in human beings.

B. Approximately 50% of calcium is free or ionized and is metabolically active.

C. Hypoalbuminemia can make the measured total calcium concentration appear artificially low.

D. Hypoventilation can decrease ionized calcium levels and in turn exacerbate symptoms of hypocalcemia.

E. Calcium is bound to citrate and is biologically inactive.

A

ANSWER: D

COMMENTS: Calcium is the most abundant cation in the human body. As much as 99% of calcium is stored in the musculoskeletal system. The remainder is present in serum and exists in three forms: (1) 45% is bound to albumin and is biologically inert, (2) 50% is ionized and metabolically active, and (3) a small percentage is complexed with citrate and is also biologically inactive.

Hypoalbuminemia means that more of the total serum calcium will be free and metabolically active. Although total serum calcium may be low, the patient may not be metabolically hypocalcemic. Ionized calcium levels are inversely affected by the pH of blood. A one-unit rise in pH will decrease the ionized calcium level by 0.36 mmol/L. Hypoventilation would cause a drop in pH and thus a subsequent rise in the ionized calcium level.

289
Q

Vitamin D synthesis begins in the skin keratinocytes. What is
the next step of activation in vitamin D synthesis?

A. Hydroxylation in the kidney to yield 1,25-dihydroxyvita- min D

B. Hydroxylation in the liver to yield 25-hydroxyvitamin D

C. Decarboxylation in the liver to yield 25-hydroxyvitamin D

D. Decarboxylation in the kidney to yield 25-hydroxyvitamin D

E. Decarboxylation in the periphery to yield 25-hydroxyvita-
min D

A

ANSWER: B
COMMENTS: Vitamin D synthesis begins in the keratinocytes of the skin. Subsequently, hydroxylation occurs in the liver to yield 25-hydroxyvitamin D. The final step in the conversion of vitamin D to its active form occurs in the kidney, where a second hydrox- ylation reaction takes place to yield 1,25-dihydroxyvitamin D. Sun- light plays a key role in the initial synthesis step in the skin. Persons who are not exposed to sunlight require supplemental vitamin D through dietary intake.

290
Q

Calcitonin helps mediate calcium homeostasis by which of the following actions?

A. Stimulates osteoblast-mediated bone formation and inhibits renal resorption of calcium and phosphate

B. Directly inhibits secretion of parathyroid hormone (PTH)

C. Inhibits intestinal absorption of calcium

D. Stimulates hydroxylation of vitamin D

E. Stimulates osteoclast-mediated bone resorption

A

ANSWER: A
COMMENTS: Calcitonin is produced by the parafollicular cells (C cells) of the thyroid gland and works to oppose the actions of PTH. It helps lower ionized calcium levels in primarily two ways. First, it inhibits osteoclast-mediated bone resorption. Second, it inhibits resorption of calcium and phosphate by the kidney. Calci- tonin has no direct effects on intestinal absorption or osteoblast- mediated bone formation. It is also used in the treatment of hypercalcemic crisis (4 IU/kg subcutaneously/intramuscularly). It acts quickly (24 to 48 h) and is more effective when used in com- bination with glucocorticoids. Finally, calcitonin is a useful marker with regard to surveillance in medullary thyroid carcinoma.

291
Q

With regard to PTH, which of the following statements is
incorrect?

A. PTH blocks calcium excretion at the ascending limb of the loop of Henle.

B. PTH stimulates osteoclast resorption of calcium and phosphate.

C. PTH cells express G protein–coupled calcium-sensing receptors.

D. PTH inhibits calcium excretion at the distal convoluted tubule of the kidney.

E. PTH enhances renally mediated hydroxylation of 25-hydroxyvitamin D.

A

ANSWER: A
COMMENTS: PTH has a variety of actions and targets to help increase serum calcium levels. The parathyroid cells express a G protein–coupled membrane receptor that senses serum calcium levels. When calcium levels fall below appropriate levels, the receptor stimulates the release of PTH into the circulation. PTH has a half-life of about 2 to 4 min in the circulation. Before its rapid clearance, it first targets osteoclasts and stimulates them to resorb calcium and phosphate from bone. In the kidney, PTH blocks calcium excretion at the distal convoluted tubule. It indirectly pro- motes intestinal absorption of calcium by enhancing the renally mediated activation of vitamin D.

292
Q

What is the cause of hypercalcemia?

A. Sarcoidosis

B. Tuberculosis

C. Histoplasmosis

D. Malignancy in hospitalized patients

E. All of the above

A

ANSWER: E

COMMENTS: A number of conditions can cause hypercalcemia, including granulomatous disorders such as sarcoidosis, tuberculo- sis, and histoplasmosis. Medications can falsely elevate the serum calcium level. Examples include thiazide diuretics, lithium, and vitamin A or D. Other conditions include Paget’s disease, immobi- lization, and malignancy. In hospitalized patients, malignancy is the most common cause of hypercalcemia.

293
Q

Routine workup of a patient with suspected primary hyperparathyroidism (PHPT) includes ruling out familial hypocalciuric hypercalcemia (FHH). Which of the following excludes FHH?

A. Nephrolithiasis

B. 24-h urine calcium

C. T-score of −3.0 at distal radius

D. Intact PTH

E. Negative sestamibi

A

ANSWER: B

COMMENTS: In a patient with suspected PHPT, it is important to document normal renal function before the interpretation of parathyroid function tests. The minimum testing that should be performed includes intact PTH, serum calcium, blood urea nitrogen (BUN), creatinine, and vitamin D levels. In addition, a 24-h urine calcium excretion should be measured to exclude FHH. Measuring PTH-related protein (PTHrp) is unnecessary unless metastatic cancer is suspected. FHH should be strongly considered in patients who have a family history of hypercalcemia and no previous history of normal calcium levels.

294
Q

All of the following are consistent with the diagnosis of
secondary hyperparathyroidism except:

A. History of chronic kidney disease (CKD)/chronic renal insufficiency (CRI)

B. Elevated serum calcium level

C. Vitamin D deficiency

D. Elevated PTH level

E. History of gastric bypass

A

ANSWER: B
COMMENTS: The pathophysiology of secondary hyperparathy- roidism is multifactorial and can result from phosphorous retention, altered vitamin D metabolism and resistance, altered metabolism of PTH, impaired calcemic response to PTH, and possible genetic mutations. The condition most commonly occurs in patients with a history of chronic renal failure. Gastric bypass has also been an increasingly recognized cause of altered vitamin D metabolism. Patients will commonly have an elevated PTH level and normal serum calcium. In such a setting, vitamin D levels should be mea- sured and, if low, treated for a minimum of 6 weeks with supple- mental vitamin D.

295
Q

All of the following are indications for surgical treatment of secondary hyperparathyroidism except:

A. Calcium phosphate product of ≤70

B. Uremic pruritus

C. Osteitis fibrosa cystica

D. Calciphylaxis

E. Tumoral calcinosis

A

ANSWER: A

COMMENTS: Secondary hyperparathyroidism is most com- monly managed medically with the use of calcimimetic agents (e.g., cinacalcet), phosphate binders, adequate calcium intake, and vitamin D replacement. Cinacalcet works by binding the calcium- sensing receptors on the chief cells of the parathyroid gland and increasing its sensitivity to extracellular calcium. Surgical treat- ment is indicated in patients with (1) renal osteodystrophy, (2) calciphylaxis, (3) calcium phosphate product of ≥70, (4) soft tissue calcium deposition and tumoral calcinosis, and (5) calcium level greater than 11 mg/dL with an inappropriately high level of PTH. Renal osteodystrophy is a major issue in hemodialysis patients. The aluminum present in the dialysate bath accumulates in the bone and contributes to the development of osteomalacia. Osteitis fibrosa cystica, a type of renal osteodystrophy, is characterized by marrow fibrosis and increased bone turnover. Bone cysts, osteopenia, and decreased bone strength develop. To halt the progression of this disease process, these patients with secondary hyperparathyroidism are treated surgically. Calciphylaxis is a rare vascular disorder in which calcium is deposited in the media of small- to medium-sized arteries. As a result, ischemic damage to the dermal and epidermal structures develops. The ulcerated lesions are extremely painful and can become infected with subsequent sepsis, and eventually death. Patients with early signs of calciphylaxis should undergo an urgent parathyroidectomy, although there is some evidence that aggressive management of serum calcium and parathyroid levels with cinacalcet may be beneficial. Care should be taken in wound care management because aggressive debridement can lead to chronic nonhealing wounds since wound healing is very poor in these patients. Uremic pruritus is characterized by severe itching that is thought to result from increased deposition of calcium salt in the dermis without the visible lesions of calciphylaxis. Parathy- roidectomy seems to alleviate these symptoms and halts progres- sion to the more serious skin and vascular complications seen with calciphylaxis.

296
Q

With regard to PHPT, which of the following statements is
true?

A. PHPT is more common in men than in women.

B. A common feature of PHPT is polyuria.

C. A history of nephrolithiasis is present in 80% of patients with PHPT.

D. Five percent of patients with PHPT can have multiple glands affected.

E. Familial hypercalcemic hypocalciuria is associated with PHPT.

A

ANSWER: B

COMMENTS: PHPT is a relatively common disorder that affects 0.3% of the human population, most commonly women. The exact cause of PHPT is unknown. In 80% of patients only a single adenoma is present, but multiple adenomas or hyperplasia can be present in up to 15%–20%. Patients with PHPT can have symp- tomatic or “asymptomatic” disease. Some degree of renal dysfunc- tion is present in up to 80% of patients. Nephrolithiasis, however, is far less common, with an incidence of approximately 20%–25%. The clinical manifestations of PHPT vary widely across patients, but if a detailed history is taken, many will complain of polydipsia and polyuria from the calciuresis associated with the disease. Although PHPT occurs sporadically in the majority of patients, in a small percentage it is part of a familial syndrome. Multiple endo- crine neoplasia type I (MEN-I) results from a germline mutation in the menin gene located on chromosome 11q12-13. Patients with MEN-I are susceptible to the development of pancreatic neuroen- docrine tumors, pituitary adenomas, and PHPT. MEN-IIA is an autosomal dominantly inherited condition caused by a germline mutation on chromosome 11 that is associated with PHPT, pheo- chromocytoma, and medullary thyroid cancer. Patients with famil- ial jaw tumor syndrome have a higher risk for the development of parathyroid carcinoma. Familial hypercalcemic hypocalciuria is associated with elevated calcium levels and low urinary excretion of calcium. The primary defect is the abnormal sensing of calcium in blood by the parathyroid gland and the renal tubules, which causes inappropriate secretion of PTH and excessive renal reab- sorption of calcium.

297
Q

In 2014, the fourth international workshop released guide- lines for the surgical management of asymptomatic PHPT, outlining indications for the surgical treatment of this disease. All of the following are part of the criteria except:

A. PHPT in a patient younger than 50 years

B. Calcium elevated to greater than 1 mg/dL above normal

C. Creatinine clearance < 60 mL/min

D. Nephrolithiasis

E. 24-h urine for calcium > 200 mg/day

A

ANSWER: E
COMMENTS: Guidelines for surgery in asymptomatic PHPT include age less than 50 years old; a serum calcium measurement 1.0 mg/dL (0.25 mmol/L) above the upper limit of normal; bone mineral density by dual-energy x-ray absorptiometry (DXA) with a T-score < −2.5 at lumbar, spine, total hip, femoral neck, or distal 1/3 radius, vertebral fracture on x-ray, computed tomography (CT), magnetic resonance imaging (MRI), 24-h urine for calcium > 400 mg/day (>10 mmol/day) and increased risk of stone on biochemical stone risk analysis, or the presence of nephrolithiasis or nephrocal- cinosis on x-ray, ultrasound, or CT. For patients with PHPT who do not undergo surgery, monitoring guidelines include at minimum obtaining an annual serum calcium, DXA every 1 to 2 years, and annual estimated glomerular filtration rate (eGFR)/serum creatinine.

298
Q

A 54-year-old woman has proximal muscle weakness, polyuria, and a depressed mood. Laboratory workup reveals a serum calcium level of 11.2 mg/dL and a PTH level of 110 ng/L. Which of the following is the least sensitive preopera- tive localization study to identify an abnormal parathyroid gland?

A. MRI

B. Single-photon emission computed tomography (SPECT)

C. Technetium-99m–labeled sestamibi scan

D. Neck ultrasound

E. Four-dimensional CT (4D-CT)

A

ANSWER: A
COMMENTS: Although all of these imaging studies have been used to identify the location of a parathyroid adenoma, MRI is the least sensitive of those listed. Routine preoperative localization in patients with PHPT includes neck ultrasound and technetium-99m– labeled sestamibi scan. Sestamibi scan has a reported sensitivity as high as 90%. Ultrasound is slightly less sensitive (75%), but the ease of in-office use makes it a useful tool for the general surgeon. SPECT, when used with planar sestamibi, is very good at locating potential ectopic glands such as those in the mediastinum. 4D-CT incorporates contrast perfusion in hyperfunctioning parathyroid tissue over time. This additional layer provides functional informa- tion in addition to the anatomic information provided by a standard CT scan. In a recent study, 4D-CT has shown improved sensitivity over all other modalities, especially when used in combination with ultrasound.

299
Q

Of the following, which patient is the least likely to have
multigland disease?

A. A 65-year-old lady with a PTH level of 110 ng/L and calcium level of 10.5 mg/dL

B. A 22-year-old woman with a PTH level of 140 ng/L, a calcium level of 10.1 mg/dL, and MEN-I

C. A 75-year-old man with a 10-year history of renal failure

D. A 44-year-old woman with a diagnosis of secondary hyperparathyroidism

E. A 39-year-old woman 6 years after a gastric bypass for morbid obesity

A

ANSWER: A
COMMENTS: No study has yet identified a reliable predictor for determining which patients with sporadic hyperparathyroidism will have multigland disease. The exception is in familial, secondary, and tertiary hyperparathyroidism. Because of the nearly uniform inci- dence of four-gland hyperplasia, all these patients are managed with bilateral neck exploration and either total parathyroidectomy with autotransplantation or three-and-a-half gland parathyroidectomy. Although some surgeons believe that patients with higher preopera- tive PTH or calcium levels (or both) are more likely to have multi- gland disease, this has not been proved to be true in clinical studies.

300
Q

A 55-year-old woman with a diagnosis of hyperparathyroid- ism wishes to undergo focused parathyroidectomy. Which of the following would preclude a patient from being a candidate for this approach?

A. PHPT

B. Lack of availability of intraoperative PTH (IOPTH)

C. Preoperative imaging of a solitary lesion on only one of two localization studies

D. Previous neck surgery

E. Secondary hyperparathyroidism

A

ANSWER: B
COMMENTS: Focused parathyroidectomy is the preferred approach in patients who have a solitary lesion that is imaged conclusively by ultrasound, sestamibi, or other appropriate imaging modalities. If the surgeon is not attempting to visualize all four glands, the use of IOPTH to determine whether all hyperfunction- ing tissue has been removed and to document an appropriate drop in PTH levels after the removal of the suspected gland is the stan- dard of care. The most common criterion used is a 50% or greater reduction in the PTH level from the baseline 10 min after parathy- roidectomy. The best clinical marker of single-gland disease is concordant preoperative imaging in combination with an appropri- ate correction of IOPTH levels. Previous neck surgery or lack of concordant imaging on two types of studies is not a contraindica- tion to attempting focused parathyroidectomy. Patients suspected of having multigland disease are managed by four-gland explora- tion via a smaller incision.

301
Q

A pregnant mother in her first trimester comes to her clinician’s office with a diagnosis of PHPT. What is the correct management?

A. Parathyroidectomy during the second trimester

B. Parathyroidectomy during the third trimester

C. Prescribing a calcimimetic agent to help reduce hypercal- cemia until after delivery, when definitive surgery can be offered safely

D. Close observation and parathyroidectomy following delivery

E. Weekly injections of calcitonin until delivery, when definitive surgery can be offered safely

A

ANSWER: A
COMMENTS: Hyperparathyroidism during pregnancy is often unrecognized and is associated with a 3.5-fold increase in miscar- riage. Loss of the pregnancy most often occurs during the late second trimester. The incidence of hyperparathyroidism in preg- nancy is 0.7%. Maternal complications include hyperemesis, neph- rolithiasis, and pancreatitis. Fetal complications include spontaneous abortion and growth retardation. In those who reach delivery, neo- natal complications include hypocalcemic crisis within the first few days of life. Calcimimetic medications have not been used in the setting of hyperparathyroidism in pregnancy. Calcitonin has no role in the management of hyperparathyroidism.

302
Q

A patient is undergoing focused exploration for PHPT. A single large parathyroid gland is found adjacent to the left superior thyroid pole. The baseline PTH level was 300 ng/L, and the pre–pedicle clamp level was 400 ng/L. Five and 10 min after the removal of the gland, the PTH level is measured to be 200 and 175 ng/L, respectively. All are appropriate next steps in the management of this patient except:

A. Repeated PTH measurement

B. Frozen-section confirmation of the removed parathyroid gland

C. Four-gland exploration

D. Drawing blood from the contralateral jugular vein to determine the PTH level

E. Conclusion of the operation given the 50% drop from the highest preremoval level

A

ANSWER: E
COMMENTS: Although this patient did experience a 50% drop from the highest preremoval level, 200 ng/L is still quite elevated. When the surgeon is going to accept a 50% drop, the final level should either approach normal or be following a kinetic trend toward normal. A preexcision level that is within a few-fold eleva- tion of the upper limit of normal should demonstrate normal kinet- ics and rapidly approach normal levels within two to three half-lives of the hormone after the adenoma is removed. The first steps in this situation should be to repeat the PTH level 15 to 20 min following gland removal and send off a frozen section for confirmation of the candidate gland removed. It would then be appropriate to either await the results of these two tests or proceed with the four-gland exploration. The use of intraoperative PTH monitoring has revolu- tionized how parathyroid surgery is performed. It has allowed sur- geons to perform directed operations and, before leaving the operating room, document whether a biochemical cure has been achieved with relative certainty. Many surgeons include determina- tion of a preincision PTH level, a stimulated (pre–pedicle ligation) level, and subsequent postremoval levels at 5-min intervals until a normal level is achieved. In patients with secondary or tertiary hyperparathyroidism and stimulation levels as high as 2000 ng/L, a 90% decrease has been suggested as being indicative of cure.

303
Q

The patient in Question 16 has a repeated PTH value of 200 ng/mL (ng/L). Frozen section of the removed left superior candidate gland shows “hyperplastic parathyroid tissue.” On further exploration, no gland can be identified in the left inferior location. The right side is explored, both glands are biopsied, and “hyperplastic parathyroid tissue” is confirmed on frozen section. Blood is drawn from the left internal jugular vein to determine the PTH level, which is found to be 600 ng/L. All of the following include appropriate steps in the management of this patient except:

A. Cervical thymectomy

B. Exploration of the left carotid sheath

C. Left thyroid lobectomy

D. Left lateral neck dissection

E. Exploration of the retroesophageal space

A

ANSWER: D
COMMENTS: The left inferior parathyroid gland has not been identified in this patient. “Hyperplastic parathyroid tissue” has been confirmed on frozen section in the three identified glands. Failure to find a normal parathyroid gland among those identified should alert the surgeon that the patient probably has four-gland hyperpla- sia. Failure to find the remaining gland places the patient at a high risk for persistent hyperparathyroidism. Most lower parathyroid glands are found in proximity to the lower thyroid pole. If not found at this location, the thyrothymic ligament and thymus should be explored. If the IOPTH level does not normalize after this maneu- ver, intraoperative ultrasound of the left thyroid lobe versus left lobectomy should be performed. Finally, central neck dissection and exploration of the left carotid sheath are performed. The pro- cedure is terminated after these steps, and further imaging and localization studies should be undertaken if the patient has persis- tent hyperparathyroidism.

304
Q

The most common site of an ectopic parathyroid gland in a
patient with persistent or recurrent hyperparathyroidism is:

A. Paraesophageal

B. Mediastinal

C. Intrathymic

D. Intrathyroidal

E. In the carotid sheath

A

ANSWER: D
COMMENTS: The most common causes for persistent and recur- rent hyperparathyroidism include ectopic parathyroids, unrecognized hyperplasia, or supernumerary glands. The most common ectopic location is paraesophageal (28%), followed by mediastinal (26%), intrathymic (25%), intrathyroidal (11%), carotid sheath (9%), and high cervical or undescended (2%). Less common causes include missed adenoma in a normal position, incomplete resection of an
abnormal gland, parathyroid carcinoma, and parathyromatosis.

305
Q

All of the following have been associated with an increased risk for hungry bone syndrome after parathyroidectomy except:

A. Graves’ disease

B. Tertiary hyperparathyroidism

C. Preoperative PTH level

D. Age

E. Large single adenomas

A

ANSWER: C
COMMENTS: “Hungry bone syndrome” is characterized by post- parathyroidectomy hypocalcemia and hypophosphatemia. Patients most at risk are those with four-gland hyperplasia from secondary or tertiary hyperparathyroidism. The postoperative calcium level in these patients can drop critically low and necessitate intravenous calcium supplementation. During this period both serum calcium and phosphate levels must be monitored closely. In some patients it can take more than 4 to 5 days for serum calcium and phosphate levels to stabilize. Other patients shown to have increased risk for this condition are those who are older or have concomitant thyro- toxicosis or a large single adenoma. The preoperative PTH level has not been found to be an independent predictor of whether “hungry bone syndrome” will develop postoperatively.

306
Q

A patient comes to the emergency department with increasing confusion, muscle weakness, nausea, vomiting, and fatigue. The serum calcium level is 16.4 mg/dL. A previously measured calcium level was 11 with a PTH of 247. The first step in the management of this patient should be:

A. Emergency parathyroidectomy

B. Aggressive intravenous hydration

C. Initiation of furosemide infusion

D. Continuous calcitonin infusion

E. Initiation of bisphosphonates

A

ANSWER: B
COMMENTS: The first step in the management of this patient with hypercalcemia should be aggressive immediate IV hydration with 0.9% saline solution and a goal urine output above 100 mL/h. After an adequate urine output is established, a loop diuretic should be given to facilitate the renal clearance of calcium. Other adjuncts include the use of bisphosphonates, calcitonin, mithramycin, gallium nitrate, and glucocorticoids. Table 17.1 lists therapies used to treat hypercalcemic crisis, along with the onset of action, advan- tages, and disadvantages of each treatment.

307
Q

Which of the following is the first sign or symptom of
hypocalcemia?

A. Shortened QT interval

B. Trousseau sign

C. Circumoral numbness

D. Anxiety

E. Laryngospasm

A

ANSWER: C

COMMENTS: Postoperative hypocalcemia following parathy- roidectomy is not unusual. The majority of patients are asymptom- atic and identified only when routine postoperative laboratory tests are obtained. In asymptomatic patients, oral calcium and vitamin D should be initiated promptly. Symptomatic patients will most com- monly complain of circumoral numbness first followed by tingling in their extremities. These two features are early signs of hypocal- cemia. PTH and calcium levels should be drawn. If the patient is hypocalcemic, high doses of calcium should be started. If PTH is undetectable, 1, 25-dihydroxy vitamin D (calcitriol) should be started 0.25 to 0.5 μg twice a day. On examination, these patients may have carpopedal spasm elicited by occlusion of blood flow to the forearm (Trousseau sign) or contraction of the facial muscles elicited by tapping on the facial nerve (Chvostek sign). However, nearly 20% of the general population has a positive Chvostek sign. If the condition is allowed to worsen, tetany, laryngeal stridor, or tonic–clonic seizures can develop, all of which can be fatal. These patients should be treated immediately with intravenous calcium.

308
Q

A 34-year-old woman has undergone three operations for hyperparathyroidism. She states that in each previous operation the surgeon has removed pathologically confirmed hyperplastic parathyroid tissue in the central portion of her neck and nodules within various muscles of her neck. Nonetheless, she continues to have elevated parathyroid levels associated with progressive bone loss. What is the most likely diagnosis in this patient?

A. Missed parathyroid adenoma

B. Multigland disease

C. Parathyromatosis

D. Familial hyperparathyroidism

E. Vitamin D deficiency

A

ANSWER: C
COMMENTS: This scenario is a classic description of a patient with parathyromatosis. This rare condition is manifested clinically as recur- rent or persistent hyperparathyroidism following multiple attempts at resection. On exploration, patients will have several small nodules of hyperfunctioning parathyroid tissue throughout the neck and possibly the mediastinum. It can be difficult to distinguish this condition from parathyroid carcinoma or an “atypical adenoma.” Parathyromatosis is believed to result from either a low-grade parathyroid malignancy, fracture of the parathyroid adenoma capsule at the original procedure, or overgrowth of embryologic rests of parathyroid tissue Management of these patients involves either serial debulking of the disease when it can be radiographically identified or pharmacologic treatment. Patients are rarely cured with surgery.

309
Q

A 54-year-old woman arrives for surgical evaluation with a PTH level of 280 ng/L and serum calcium level of 14.5 mg/ dL. She has a past medical history of mild renal failure (creatinine, 1.9 mg/dL) and jaw tumor syndrome. The most likely diagnosis in this patient is:

A. Parathyromatosis

B. Missed adenoma

C. Parathyroid carcinoma

D. Secondary hyperparathyroidism

E. Tertiary hyperparathyroidism

A

ANSWER: C
COMMENTS: A preoperative serum calcium level greater than 14 mg/dL, a palpable nodule, and adherence to surrounding tissues have all been found to be useful predictors of parathyroid carci- noma. Additionally, patients with familial hyperparathyroidism with jaw tumor syndrome, which is associated with tumor suppres- sor locus HRPT2 (parafibromin or CDC73) on chromosome 1, have an increased risk for parathyroid cancer. Parathyroid carcinoma is rare and represents just 1% of all cases of PHPT. The prognosis is extremely poor, and most patients have advanced disease when initially seen. Parathyroid carcinoma should be suspected in a patient with a preoperative calcium level greater than 14 mg/dL or a serum PTH level two to three times the upper limit of normal. Intraoperatively, parathyroid cancers are typically large and less brown than a benign parathyroid adenoma. Invasion or adherence to the surrounding tissues should also raise suspicion for parathy- roid carcinoma. Appropriate surgical management of this condition entails en bloc resection of the tumor with ipsilateral thyroid lobec- tomy. It is not necessary to perform a total thyroidectomy. A com- partment-oriented lymph node dissection is necessary only if there is gross evidence of lymph node involvement.

310
Q

Which of the following is not part of the standard workup of
a patient with persistent or recurrent hyperparathyroidism?

A. Selective venous sampling

B. SPECT

C. Technetium-Tc-99m-sestamibi scanning

D. Ultrasound

E. Positron emission tomography (PET)

A

ANSWER: E

COMMENTS: Persistent hypercalcemia is characterized by hypercalcemia that fails to correct after parathyroidectomy. Recur- rent hyperparathyroidism occurs less frequently and is defined by the development of postoperative hypercalcemia at least 6 months after having achieved normocalcemia. Preoperative localization is the mainstay of treatment algorithms for patients with persistent or recurrent hyperparathyroidism. The tests available can be divided into those that are noninvasive versus invasive. The noninvasive tests include ultrasound, sestamibi scanning, SPECT, MRI, stan- dard CT, and 4D-CT. Invasive testing includes selective venous sampling and intraoperative radioprobe guidance. Patients receive intravenous technetium-99m in the morning of the surgery, and then the radioprobe is used intraoperatively to identify the parathy- roid gland. One limitation of this technology is a high background level, which can prevent the identification of an intrathyroidal para- thyroid gland. PET has yet to have any defined role in parathyroid localization.

311
Q

Which of the following statements regarding the anatomy of the adrenal glands is incorrect?

A. They are retroperitoneal organs.

B. The right adrenal gland is semilunar in shape.

C. They are yellow or sulfur in appearance.

D. The left adrenal is closely associated with the spleen and tail of the pancreas.

E. The adrenal medulla is derived from the neural crest tissue.

A

ANSWER: B
COMMENTS: The adrenal glands are retroperitoneal organs located superior and medial to the kidneys. The right gland is pyramidal in shape (resembling a Witch’s hat), and the left gland is semilunar in shape. The adrenal gland is divided into two parts, a cortex and a medulla, which account for approximately 80%–90% and 10%–20% of the gland’s volume, respectively. Histologically, the cortex is divided into the outer zona glomerulosa, the middle zona fasciculata, and the inner zona reticularis. The adrenal medulla is composed of chromaffin cells derived from the neural crest tissue.

312
Q

In regard to the vascular anatomy of the adrenal gland, which of the following statements is incorrect?

A. The superior arterial supply arises from branches of the inferior phrenic artery.

B. The left adrenal vein, joined by the inferior phrenic vein, drains into the left renal vein.

C. The right adrenal vein enters directly into the inferior vena cava (IVC).

D. The inferior arterial supply arises from branches originating from the aorta.

E. The adrenal vein is longer on the left side.

A

ANSWER: D
COMMENTS: The arterial supply of the adrenal gland arises from three sources. Superiorly, branches arise from the inferior phrenic artery, whereas the middle branches originate from the aorta. Along the medial and inferior aspect of the gland are contributory branches given off by the ipsilateral renal artery. The anatomy of the venous drainage of the left and right adrenal gland differ. The inferior phrenic vein joins the left adrenal vein before it drains into the left renal vein and can measure 2 cm in length. Variably, the left inferior phrenic vein will drain separately into the left renal vein. On the right, the adrenal vein enters directly into the IVC posteriorly and is shorter and broader than the left one. This shorter configuration can make ligation technically more challenging.

313
Q

What is the rate-limiting enzyme of catecholamine synthesis?

A. Tyrosine hydroxylase

B. Monoamine oxidase

C. Dopamine β-hydroxylase

D. Dopa decarboxylase

E. Phenylethanolamine-N-methyltransferase (PNMT)

A

ANSWER: A

COMMENTS: Tyrosine hydroxylase is the rate-limiting enzyme in catecholamine synthesis. The first step in catecholamine synthe- sis involves the conversion of l-tyrosine into dihydroxyphenylala- nine (l-dopa) by the enzyme tyrosine hydroxylase. Dopa decarboxylase then converts l-dopa into dopamine. Dopamine is subsequently converted to norepinephrine by the enzyme dopamine α-hydroxylase. PNMT is the enzyme located exclusively in the adrenal gland and is responsible for the conversion of norepineph- rine into epinephrine.

314
Q

What enzyme is responsible for the conversion of norepi-
nephrine to epinephrine?

A. Tyrosine hydroxylase

B. Monoamine oxidase

C. Catechol O-methyltransferase

D. PNMT

E. Dopamine β-hydroxylase

A

ANSWER: D
COMMENTS: Histologically, the adrenal gland is divided into two components: the centrally located medulla and the peripherally located cortex. The adrenal cortex arises from the mesoderm and accounts for approximately 90% of the total adrenal mass. Histo- logically, the cortex is made up of three zones: the glomerulosa, fasciculata, and reticularis. Each zone corresponds to the synthesis of mineralocorticosteroids, corticosteroids, and sex steroids, respectively. The medulla is composed of chromaffin cells derived from ectodermal neural crest cells. The chromaffin cells are inner- vated by sympathetic fibers traveling from the sympathetic chain. They secrete the vasoactive catecholamines, epinephrine and nor- epinephrine. Norepinephrine is converted to epinephrine by the enzyme PNMT. This enzyme is exclusively located within the adrenal medulla and is not found in ectopic adrenal medullary tissue. Thus ectopic pheochromocytomas are incapable of produc- ing epinephrine since they lack this enzyme.

315
Q

Aldosterone secretion is under the control of all of the
following except:

A. Potassium

B. Adrenocorticotropic hormone (ACTH)

C. Angiotensin II

D. Heparin

E. Epinephrine

A

ANSWER: E
COMMENTS: Aldosterone synthesis is under the control of angio- tensin II, potassium, and, to a lesser extent, ACTH. Its synthesis is inhibited by somatostatin, dopamine, atrial natriuretic factor, and heparin. Aldosterone synthase (CYP11B2) is restricted to the zona glomerulosa, where aldosterone is primarily synthesized. Angioten- sin II and potassium stimulate aldosterone secretion by increasing the transcription of CYP11B2. ACTH increases aldosterone secre- tion by no more than 10%–20% over baseline values and does so by stimulating the earlier pathways of adrenal steroidogenesis. Epi- nephrine plays no direct regulatory role in aldosterone synthesis.

316
Q

Congenital adrenal hyperplasia (CAH) is most commonly
caused by a deficiency of which of the following enzymes?

A. 21-Hydroxylase

B. 17α-Hydroxylase

C. 11β-Hydroxylase

D. 5α-Reductase

E. 21β-Hydroxylase

A

ANSWER: A
COMMENTS: The most common cause of CAH is mutations or deletions of CYP21A2, the gene that encodes for 21-hydroxylase. This enzyme defect accounts for more than 90% of the cases of CAH. In most patients, it is manifested as a salt-wasting form in which lack of the 21-hydroxylase enzyme impedes the downstream synthesis of aldosterone. Clinically, this becomes apparent within the first few months of life with the development of hypovolemia and hyperkalemia. There is excess production of ACTH because of the lack of negative feedback on steroid synthesis. Upstream pre- cursors accumulate as a result of lack of the 21-hydroxylase enzyme and are then shunted into the sex steroidogenesis pathway. This leads to the presence of ambiguous genitalia in females. The diag- nosis is made by finding elevated levels of 17-hydroxyprogeste- rone, the 21-hydroxylase substrate, and by genetic testing.
Re

317
Q

A 75-year-old male patient is currently being treated in the surgical intensive care unit for urosepsis following partial colectomy. His laboratory values are as follows: hemoglobin, 8 g/dL; white blood cell count, 8 × 103 cells/μL; and blood glucose, 34 mg/dL. In postoperative week 2, increasing pressor requirements develop suddenly in this patient, whose sepsis had been resolving despite broadening his antibiotic coverage and blood cultures being negative. You suspect adrenal insufficiency. What is the most definitive diagnostic test?

A. Serum chemistry panel

B. Morning serum cortisol

C. Cosyntropin stimulation test

D. Morning salivary cortisol

E. Mixed venous oxygen saturation

A

ANSWER: C
COMMENTS: Acute adrenal insufficiency is a life-threatening emergency. In the critically ill patient population it can develop in either the acute or chronic phase of the illness. In the intensive care setting, if acute hypotension that is not cardiogenic in origin and is refractory to pressor support develops suddenly, acute adrenal insufficiency should be excluded. The cosyntropin stimulation test is the definitive means of diagnosing adrenal insufficiency. This test, however, can take up to 24 h to return. In the acute setting, evidence of hyperkalemia, hyperglycemia, and refractory hypoten- sion is sufficient to begin empirically treating these patients with steroids until the diagnosis can be confirmed. Morning salivary cortisol concentration higher than 5.8 ng/mL or morning serum cortisol concentration higher than 15 ug/dL rule out adrenal insuf- ficiency as the cause.

318
Q

The patient described in Question 7 has worsening hypoten- sion despite an escalation in intravenous (IV) norepinephrine. What is the best next step in the management of this patient?

A. Addition of vasopressin

B. IV fluids

C. Cosyntropin stimulation test

D. Hydrocortisone injection

E. Blood transfusion

A

ANSWER: B
COMMENTS: The first step in the management of a hypotensive patient suspected of having adrenal insufficiency is volume resus- citation followed by empirical steroid replacement. Corticosteroid replacement in patients with acute adrenal insufficiency should include the IV administration of either hydrocortisone 100 mg every 6 to 8 h or dexamethasone 4 mg every 24 h. Dexamethasone is long acting and does not interfere with the administration of a cosyntropin stimulation test. Blood transfusion will not reverse this patient’s condition and is not generally recommended for patients with a hemoglobin level greater than 7 g/dL.

319
Q

A 30-year-old female with type I diabetes presents with nausea, vague abdominal pain, and skin hyperpigmentation. Important principles in the management of this disease include all of the following except:

A. Mimic endogenous rhythm

B. Simple monitoring and dose titration

C. Use standard treatment dose

D. Minimize the risk of overtreatment

E. Treat with glucocorticoids/mineralocorticoids

A

ANSWER: C
COMMENTS: In 1855, Thomas Addison described a clinical con- dition that involved salt wasting, skin hyperpigmentation, and his- topathologic destruction of the adrenal gland. Additional clinical symptoms of the disorder can include nausea and vague abdominal pain, musculoskeletal complaints, and postural dizziness. The most characteristic feature of Addison disease is hyperpigmentation of the skin and mucous membranes. When Addison disease was first described, tuberculous adrenalitis was the most common cause of primary adrenal insufficiency. As a result of the decreased inci- dence of advanced tuberculosis, the current most common cause is an autoimmune reaction known as Addison disease. Autoimmune Addison disease is typically associated with other autoimmune disorders such as type 1 diabetes mellitus.
In patients with chronic adrenal insufficiency, maintenance therapy can be achieved with short-acting glucocorticoids (e.g., hydrocortisone—multiple recommended regimens), which are pre- ferred as they better mimic the normal diurnal rhythm, longer-acting glucocorticoids (e.g., prednisone), which may be useful in noncom- pliant patients, and the mineralocorticoid fludrocortisone, usual dose 0.1 mg/day. Treatment regimens should be tailored to each individual patient and monitored on a regular basis. Supplemental corticoste- roids are required during physiologic stress such as illness, trauma, anesthesia, and surgical procedures. One standard dose should not be applied to all patients with adrenal insufficiency, but should rather be individualized to account for the amount of glucocorticoids the patient is taking and the procedure or stress they are undergoing.

320
Q

The most common endogenous cause of Cushing syndrome is:

A. Adrenocortical carcinoma (ACC)

B. ACTH-hypersecreting pituitary adenoma

C. Cortisol-hypersecreting adrenal adenoma

D. Ectopic ACTH-producing tumor

E. Adrenal hyperplasia

A

ANSWER: B
COMMENTS: Overall, the most common cause of Cushing syn- drome is the exogenous use of corticosteroids. Among the endog- enous types of Cushing syndrome, an ACTH-hypersecreting pituitary adenoma is the most common (70%). Adrenal adenomas and ectopic ACTH-producing tumors each account for 10% of the endogenous causes of Cushing syndrome.

321
Q

A 58-year-old man is involved in a motor vehicle accident and presents to the trauma bay with a Glasgow Coma Scale score of 7 for which he is intubated. Use of which pharmaco- logic agent can lead to adrenal insufficiency?

A. Succinylcholine

B. Propofol

C. Ketamine

D. Midazolam

E. Etomidate

A

ANSWER: E

COMMENTS: Etomidate, a commonly used induction agent for general anesthesia, is a potent inhibitor of the 11-hydroxylase enzyme. It is the only inhibitor of steroid synthesis that can be given parenterally and can lead to the development of adrenal insuffi- ciency. It is the treatment of choice in critically ill patients with hypercortisolism who are unable to take oral medications. Its onset of action is very rapid (<1 min), and its half-life is only 3 to 5 h. The medication is administered as a continuous infusion, with nonhypnotic doses ranging between 0.2 and 0.6 mg/kg per h. The infusion is titrated according to the decline in serum cortisol levels. Once the cortisol levels are brought within a physiologic range, some patients may require hydrocortisone supplementation to avoid the development of adrenal insufficiency.

322
Q

On workup, a patient is found to have elevated free cortisol and plasma ACTH levels. Further testing reveals that both low- and high-dose dexamethasone administration fail to suppress cortisol production. What is the most likely diagnosis?

A. Bilateral adrenal hyperplasia

B. Pituitary tumor

C. Adrenal adenoma

D. Ectopic ACTH-producing tumor

E. Exogenous corticosteroids

A

ANSWER: D
COMMENTS: Failure to suppress the cortisol production after the administration of high-dose dexamethasone concomitant with ele- vated plasma ACTH levels suggests that the hypothalamic-pitu- itary-adrenal axis is not intact. This scenario suggests the presence of an ectopic source of ACTH. Ectopic ACTH-producing tumors account for 10%–15% of cases of Cushing syndrome. Thoracic tumors are more common than abdominal tumors as a cause of ectopic ACTH syndrome. Of those arising in the mediastinum, primary lung carcinoids are the most common. Following in fre- quency are small cell lung cancer and thymic tumors. In approxi- mately 20% of patients, the site of ACTH production is never found.

323
Q

A 26-year-old female with a history of deep vein thromboses presents to the office with hypertension, facial plethora, and truncal obesity. Which of the following additional features is not associated with this syndrome?

A. Hyperglycemia

B. Decreased libido

C. Hyperkalemia

D. Nephrolithiasis

E. Proximal muscle weakness

A

ANSWER: C
COMMENTS: Patients with Cushing syndrome can have a wide variety of symptoms. Classically, patients will have newly diag- nosed or poorly controlled hypertension, glucose intolerance, and truncal obesity. Additional signs and symptoms include easy bruising, proximal muscle weakness, decreased libido, nephroli- thiasis, and a flushed facial appearance (facial plethora). Nephro- lithiasis occurs in up to 50% of patients with Cushing syndrome. The underlying pathogenesis is not yet clearly defined. There is evidence that patients with Cushing syndrome have elevated urinary uric acid secretion, which could contribute to increased stone formation. Additionally, there is a documented increased risk of 1.9% for deep venous thrombosis in patients with Cushing syndrome in the nonoperative setting. In postoperative Cushing patients, the risk for venous thromboembolism ranges between 0% and 5.6%. Chronic glucocorticoid excess produces a metabolic syndrome that is associated with increased morbidity and mortal- ity. The elevated risk for venous thromboembolic disease is thought, in part, to be due to the increased prevalence of cardio- vascular disease, glucose intolerance, and obesity. Some research- ers have postulated that hypercortisolism is associated with a hypercoagulable state independent of these associated risk factors. Hyperkalemia is not a feature of Cushing syndrome; in fact, hypo- kalemia is more likely to occur given the weak mineralocorticoid effect of cortisol.

324
Q

A 37-year-old-female presents to the emergency depart- ment with complaints of abdominal pain and weight loss. On physical examination, she is found to have facial plethora, purple striae, and truncal obesity. Computed tomography (CT) scan reveals a 5.7-cm right adrenal mass with invasion into the IVC. There are no demonstrable sites of metastases. Which of the following is the best treatment option?

A. Neoadjuvant therapy with mitotane

B. Laparoscopic right adrenalectomy

C. Right adrenal biopsy

D. Observation

E. Open right adrenalectomy with tumor thrombus extirpa- tion and IVC reconstruction

A

ANSWER: E
COMMENTS: Adrenal cortical carcinomas are rare with a worldwide incidence of 2 per 1 million. Size is the most impor- tant criteria to help diagnose malignancy. The majority are spo- radic, but they are also associated with Li-Fraumeni syndrome and multiple endocrine neoplasia type 1. Approximately half are nonfunctioning, while the other half secretes cortisol (30%), androgens (20%), estrogens (10%), aldosterone (2%), and mul- tiple hormones (35%). Adequacy of resection is the most impor- tant predictor of survival. Those who undergo complete en bloc resection with involved lymph nodes and other surrounding organs (e.g., diaphragm, kidney, pancreas, liver, or IVC) have 5-year survival rates ranging from 32% to 48%. Mitotane has adrenolytic activity and can be used for the treatment of meta- static or unresectable disease. Systemic chemotherapeutic agents can also be used but provide inconsistent responses. There is a role for neoadjuvant therapy in those with borderline resectable cases.

325
Q

Which of the following is least likely to metastasize to the
adrenal glands?

A. Pancreatic tumors

B. Thyroid carcinoma

C. Lung carcinoma

D. Breast carcinoma

E. Melanoma

A

ANSWER: B
COMMENTS: The rich vascular supply of the adrenal glands predisposes them to metastases. Approximately 25% of patients with carcinomas will eventually develop adrenal involvement. The primary cancers are those of the gastrointestinal tract, lung, breast, pancreas, kidney, and skin (melanoma). It remains essential to complete a functional workup for adrenal incidentalomas even in a patient with a prior history of malignancies. If, however, the functional workup is negative and clinical suspicion is high for a metastatic adrenal lesion, in contrast to primary adrenal lesions, biopsy of the adrenal gland may be warranted to aid in establishing a diagnosis.

326
Q

A 13-year-old female presents to the office complaining of frequent headaches, anxiety, and facial flushing. Her family history is significant for acquired blindness. Which of the following conditions is not associated with her disease process?

A. Angiomatosis

B. Hypoglycemia

C. Renal cell carcinoma

D. Hemangioblastomas

E. Pheochromocytoma

A

ANSWER: B
COMMENTS: This patient has Von Hippel-Lindau disease with an associated pheochromocytoma. Pallor, headache, and a sense of impending doom are all relatively common symptoms of patients with pheochromocytoma. Hyperglycemia is also a relatively common feature in such patients. The insulin-producing islet cells of the pancreas are under inhibitory control by α2-receptors. Thus with catecholamine excess, a relative hypoinsulinemia can develop and lead to hyperglycemia. Flushing, nausea, and fever can occur rarely.
Elevated plasma-free metanephrine and normetanephrine levels are the most sensitive biochemical tests to diagnose a pheochromocytoma. However, its specificity (risk for false-posi- tive results) is lower than that of urinary epinephrine, urinary norepinephrine, urinary vanillylmandelic acid, and urinary total metanephrines. The specificity of plasma-free metanephrines can be improved by using a cutoff value of at least four times the upper limit of normal. Table 18.1 illustrates the specificity of other biochemical tests performed for the evaluation of pheochromocytoma.

327
Q

The initial biochemical screening tests for incidentally discovered adrenal nodules include all of the following except:

A. High-dose dexamethasone test

B. Plasma aldosterone concentration (PAC)

C. Low-dose dexamethasone test

D. Plasma renin activity (PRA)

E. Plasma-free catecholamines

A

ANSWER: A
COMMENTS: The workup of an incidental adrenal mass includes screening and secondary confirmatory testing. Screening tests include (1) a low-dose dexamethasone test or a 24-h urine cortisol (or both), (2) a morning PAC and PRA, (3) plasma-free metaneph- rine and normetanephrine or 24-h total and fractionated urinary metanephrines (or both), and (4) a late-night salivary cortisol test. Confirmatory tests are done when the results of initial testing are equivocal and include, but are not limited to, a high-dose dexa- methasone test and a salt-loading aldosterone suppression test. Measurement of the serum ACTH level is performed once a diag- nosis of hypercortisolism is established. Cortisol levels follow a circadian rhythm, which explains why random cortisol levels are not useful in the screening of these patients. When there is normal diurnal variation, cortisol should be at its lowest at late night. In patients suspected of having Cushing syndrome, an elevated serum cortisol level at 11 pm can be an early, albeit not definitive, factor in the diagnosis of the condition. For its simplicity, measurement of late-night salivary cortisol levels has increased in popularity and has a sensitivity and specificity approaching 90%–95%. A low or suppressed level of ACTH or dehydroepiandrosterone sulfate (DHEAS) concentration further supports the diagnosis of subclini- cal Cushing syndrome. In recent years, DHEAS has been obtained when there is a suspicion for ACC as well, as this can be elevated.

328
Q

A 43-year-old woman has a 5-year history of poorly controlled hypertension, fatigue, and myalgias. She is taking three different medications, including a diuretic and β-blocker. What is the next best step in establishing the diagnosis of surgically correctable hypertension?

A. CT of the abdomen and pelvis

B. 24-h total and fractionated urinary metanephrines

C. PAC and PRA

D. Renal ultrasound

E. Saline suppression testing

A

ANSWER: C

COMMENTS: The most likely diagnosis in this patient is primary hyperaldosteronism. There is a general consensus that all patients with young age, poorly controlled hypertension, and a history of hypokalemia should undergo evaluation for an aldoste- rone-secreting adenoma. Hypokalemia can be associated with myalgias, constipation, and fatigue. The initial step is measuring the aldosterone-to-renin ratio. This test should be performed after discontinuing such interfering medications as spironolactone. An aldosterone-to-renin ratio of greater than 20 (e.g., a serum aldo- sterone level of ≥10 ng/dL with a renin level of <0.5 ng/mL) is suggestive of the diagnosis. The diagnosis is then confirmed by doing a 24-h urine aldosterone, sodium, and potassium test with the patient ingesting a high-salt diet. A 24-h urinary aldosterone level greater than 12 μg in 24 h is considered positive. After establishing the biochemical diagnosis, the next step is to deter- mine whether there is laterality of the disease. Only at this time should diagnostic imaging be ordered.

329
Q

Biochemical testing confirms the diagnosis of primary hyperaldosteronism in the patient in Question 18. The next step in the management should be:

A. Imaging repeated in 6 months

B. Long-term management with spironolactone

C. Bilateral cortical-sparing adrenalectomies

D. Selective venous sampling

E. Laparoscopic ultrasound

A

ANSWER: D
COMMENTS: After establishing the biochemical diagnosis, the next step is to determine whether there is laterality of the disease. Selective venous sampling is indicated when patients are >40 years old, have bilateral adrenal lesions, or no visible adrenal lesion on imaging. Adrenal vein sampling (AVS) is done by performing simultaneous measurements of serum cortisol and aldosterone in the cannulated adrenal veins and the peripheral circulation. Confir- mation of successful cannulation is established by documenting a greater than five-fold elevation in cortisol concentration relative to the peripheral circulation. Lateralization is confirmed by an unbal- anced ratio of aldosterone to cortisol when comparing one side with the other. Corrected aldosterone/cortisol ratios (i.e., aldosterone/ cortisol ratio of one side to aldosterone/cortisol ratio of the other side) of greater than 4:1 are indicative of a unilateral source of aldosterone excess.

330
Q

The patient in Question 18 undergoes diagnostic imaging.
What is the most likely finding on imaging?

A. Bilateral idiopathic adrenal hyperplasia

B. Unilateral adenoma

C. Normal-appearing adrenal glands

D. Bilateral adrenal adenomas

E. Unilateral adrenal hyperplasia

A

ANSWER: A
COMMENTS: The most common cause of primary hyperaldos- teronism is bilateral idiopathic hyperplasia (60%–70%). The second most common cause is an aldosterone-producing adenoma (35%), also known as Conn’s syndrome. The remaining subtypes include unilateral adrenal hyperplasia (2%), carcinoma (<1%), and familial hyperaldosteronism types I and II (<1%). It is important to distin- guish which subtype of hyperaldosteronism a patient has because some types, including bilateral idiopathic hyperplasia, are managed nonoperatively.
Secondary hyperaldosteronism is the result of increased renin production by the kidney. This condition can be the consequence of reduced intravascular volume as is seen in congestive heart failure, cirrhosis, nephrosis, renovascular hypertension, Bartter syndrome, and pregnancy. Adrenal function is normal, and treat- ment is directed at the underlying condition.

331
Q

A 55-year-old male smoker with no previous medical history comes to the surgical clinic after having recently undergone CT for nonspecific abdominal pain. A 2.5-cm left adrenal mass was identified on CT. What is the next step in the management of the incidental adrenal mass in this patient?

A. Magnetic resonance imaging (MRI)

B. CT-guided fine-needle aspiration (FNA) biopsy

C. Biochemical workup

D. Observation and follow-up CT in 6 months

E. CT of the lung

A

ANSWER: C
COMMENTS: Despite no previous history suggesting an underly- ing biochemical syndrome, the first step in the management of an incidental adrenal lesion is to complete a functional evaluation. If the lesion is nonfunctional, the next step in the management is to distinguish benign from malignant disease. MRI can be useful in this regard, but most times an index of suspicion for a malignant process can be garnered from CT alone. An image-guided biopsy is not believed to be useful in helping differentiate benign from malignant adrenal lesions. According to the National Institutes of Health State-of-the-Science Conference on the workup of adrenal incidentalomas, FNA is recommended in patients with a history of malignancy and no other signs of metastases. Before percutaneous biopsy, it is imperative that the presence of a pheochromocytoma be excluded since a life-threatening pheochromocytoma crisis can occur with this intervention. Metastatic lung cancer should be con- sidered in the differential diagnosis, but CT of the lung should not be the initial step in this patient.

332
Q

A 66-year-old woman has a 3.8-cm left adrenal mass noted on CT. On review of the CT scan, which of the following suggests a benign lesion?

A. Mass larger than 4 cm

B. Contrast washout of less than 50% at 10 min

C. Unilateral lesion

D. Heterogeneous enhancement

E. Less than 10 Hounsfield units (HU)

A

ANSWER: E
COMMENTS: In the evaluation of nonfunctional tumors, there are a number of radiographic findings that can assist the clinician in determining whether a lesion is likely to be benign or malignant (Table 18.2). Measurement of HU on CT is a commonly used parameter and is associated with acceptable rates of specificity and sensitivity when other features such as size, shape, or growth are considered. Less than 10 HU on noncontrast CT is indicative of a benign lesion. When IV contrast material is administered and delayed imaging is performed, the rapidity of washout of the con- trast agent can be measured. Benign lesions typically demonstrate rapid washout, with a greater than 50% loss of initial attenuation on delayed imaging at 10 to 15 min. Less than 50% washout sug- gests malignancy or pheochromocytoma. Lipid-poor adenomas may have values of 20 to 40 HU on noncontrast CT but will dem- onstrate >50% washout at 10 to 15 min.

333
Q

. The patient in Question 22 undergoes repeat CT in 1 year, followed by MRI. CT shows the lesion has grown from 3.8 to 4.1 cm during that period. Characteristics on CT reveal a low-attenuation lesion of less than 10 HU. MRI shows a hyperintense lesion on T1-weighted in-phase MRI. What is the probable diagnosis in this patient?

A. Adrenocortical adenoma

B. Pheochromocytoma

C. ACC

D. Myelolipoma

E. Aldosteronoma

A

ANSWER: D
COMMENTS: The imaging characteristics described are typical of myelolipoma. These lesions are composed of erythroid, myeloid, and an abundant amount of adipose tissue. On noncontrast CT, they are of low attenuation and consist of nearly pure fat. On MRI, they are hyperintense on T1-weighted images. On T2-weighted images, malignant lesions are typically hyperintense, as are pheochromo- cytomas, while benign lesions are isointense relative to the liver. In comparison, aldosteronomas rarely get this large and are slower growing. Myelolipomas are not indicated for surgical removal unless they are causing symptoms of compression or pain.

334
Q

According to the most current recommendations, at what size should operative intervention of adrenal incidentalomas be considered?

A. 2 cm 
B. 3 cm 
C. 4 cm 
D. 5 cm 
E. 6 cm
A

ANSWER: C
COMMENTS: The American Association of Clinical Endocri- nologists (AACE)/American Association of Endocrine Surgeons (AAES) medical guidelines for the management of adrenal inci- dentalomas recommend that surgical resection be considered for any nonfunctional adrenal incidentaloma 4 cm or larger. Observa- tion is appropriate for lesions smaller than 4 cm. In distinguishing between a malignant tumor and a benign lesion, size alone was associated with a 93% sensitivity and 24% specificity using the cutoff of 4 cm. With improved imaging and the use of laparoscopy in adrenal surgery, more benign tumors are being removed.

335
Q

A 22-year-old man with a history of multiple endocrine neoplasia type IIA (MEN-IIA) is found on biochemical surveillance to have elevated serum catecholamines. CT reveals a 1-cm left adrenal mass and right adrenal fullness, but no discrete mass. What is not an appropriate management of this patient?

A. Bilateral adrenalectomy

B. Left adrenalectomy with a cortical-sparing right adrenalectomy

C. Left adrenalectomy

D. Bilateral cortical-sparing adrenalectomy

E. Cortical-sparing left adrenalectomy

A

ANSWER: A
COMMENTS: Considerable controversy exists around the appro- priate surgical management of pheochromocytomas in patients with MEN. If at initial evaluation only one side shows evidence of a mass and a unilateral adrenalectomy is performed, the chance of a contralateral pheochromocytoma developing over an interval of 12 years is 52%. Patients with MEN type II should not undergo pro- phylactic removal of the contralateral side if no mass or presence of pheochromocytoma is confirmed. When the laterality of the pheochromocytoma is at question and the patient’s disease is rela- tively asymptomatic, some surgeons favor observation in young patients as nearly a fourth of patients will experience at least one episode of acute adrenal insufficiency requiring hospitalization. This is a special concern in young patients, in whom the reliability of taking medications regularly or adjusting for physiologic stress- ors is highly variable. Alternatively, surgeons have begun perform- ing laparoscopic cortical-sparing procedures in these patients, in whom the success rate of avoiding exogenous steroid dependence is reported to be between 65% and 100%. Thus in the patient described here, it would be appropriate to perform a left cortical- sparing adrenalectomy, a left adrenalectomy, a left adrenalectomy with the cortical-sparing removal of the right, or a bilateral cortical- sparing adrenalectomy. AVS in these patients preoperatively can be a useful adjunct to assess the prevalence of bilateral pheochromo- cytomas when imaging is not able to lateralize the side of an active disease.

336
Q

A 44-year-old man is due to undergo a laparoscopic adrenal- ectomy for a pheochromocytoma. Which of the following is not an appropriate optimization strategy prior to surgery?

A. α-Blockade

B. Dysrhythmia control with medications

C. Calcium channel blockers with plan for IV magnesium infusion intraoperatively

D. Cardioversion of arrhythmia

E. β-Blockade as needed

A

ANSWER: D
COMMENTS: Preoperative catecholamine receptor blockade is done to reduce the incidence and magnitude of intraoperative fluctuations in blood pressure and the development of arrhyth- mias. A α-blocker such as atenolol should never be administered before the use of an α-blocker because α-blockade can cause severe vasoconstriction and hypertension from inhibition of the vasodilator action of epinephrine. Prazosin and phenoxybenza- mine are both α-blockers and have been used routinely for pre- operative α-blockade in patients with pheochromocytoma. The use of calcium channel blockers has become more popular as they are better tolerated by patients, readily available, and less costly, and can be used as a substitute or as an adjunct to an α-blocker. Intraoperative magnesium sulfate infusion is used for its cardiovascular-stabilizing properties. The benefit of these agents is that coronary vasospasm is reduced and, when given in conjunction with an α-blocker, they can reduce the dosage of the α-blocker needed.

337
Q

The patient in Question 26 demonstrates signs of confusion and complains of sweating and headache several hours following his operation. His blood pressure is 130/65 mmHg, heart rate is 85 beats/min, and respiratory rate is 12 breaths/min. What is the most likely cause of his symptoms?

A. Dehydration

B. Postoperative bleeding

C. Hypoglycemia

D. Narcotic overdose

E. Incomplete removal of the pheochromocytoma

A

ANSWER: C
COMMENTS: In the postoperative setting, patients with pheo- chromocytoma should be monitored closely for signs or symp- toms associated with hypoglycemia. Profound hypoglycemia can develop in these patients as a result of the rebound hyper- insulinemia that occurs with the removal of the inhibitory cat- echolamine effect. Liver glycogen stores may be severely depleted in these patients because catecholamines promote gly- cogen breakdown. Thus the patient’s ability to respond acutely to the hypoglycemia is impaired. It is unpredictable in which patient the hypoglycemia will develop, so those with pheochro- mocytoma are usually administered a prophylactic dextrose infusion postoperatively.

338
Q

A 22-year-old student is scheduled to undergo parathyroidectomy for hyperparathryoidism associated with familial multiglandular syndrome. His sister developed peptic ulcer disease secondary to a Zollinger-Ellison (hypergastrinemia) tumor of the pancreas. On examination, a swelling was noted over the posterior aspect of the patient’s fifth rib. What is the most likely finding?

a
Metastasis from a parathyroid carcinoma

b
Osteitis fibrosa cystica (brown tumor) and subperiosteal resorption of the phalanges

c
Dermoid cyst

d
Eosinophilic granuloma

e
Chondroma

A

Patients with hyperparathyroidism develop demineralization, and 1.5% shows osteitis fibrosa cystica.

The presence of subperiosteal resorption of bone of the phalanges and lamina dura of the teeth are fairly diagnostic radiological findings of hyperparathyroidism.

Chondromas account for 20% of benign tumors of the rib and occur at the costochondral junction.

Osteochondromas arise from the cortex and usually occur in men.

Eosinophilic granuloma results in a destructive lesion apparent on x-ray.

339
Q

What two things do you need to rule out before diagnosing primary hyperparathyroidism?

A

Low vitamin D levels
Familial hypocalciuric hypercalcemia

Primary hyperparathyroidism (PHPT) is the result of inappropriate parathyroid hormone production by one or more intrinsically abnormal parathyroid glands and is the most common cause of hypercalcemia.

Parathyroid carcinoma is responsible for approximately 1% of PHPT and generally is characterized by very high blood calcium and parathyroid hormone levels (total serum calcium >14 mg/dL, with blood parathyroid hormone levels >300 pg/mL).

The diagnosis of PHPT is entirely biochemical and depends on the presence of both inappropriate blood calcium and parathyroid hormone level elevations, in the context of normal blood vitamin D levels.

Vitamin D plays an important role in normal blood calcium homeostasis, and low vitamin D levels may result in an appropriate compensatory elevation in blood parathyroid hormone, leading to an incorrect diagnosis of PHPT.

In addition, biochemical testing for familial hypocalciuric hypercalcemia (FHH), an autosomal dominant syndrome resulting from loss-of-function mutation(s) in the calcium sensing receptor gene, is necessary before diagnosing PHPT. This condition is characterized by mild elevations in blood-intact parathyroid hormone level, with associated relative hypercalcemia, and is diagnosed by 24-hour urine calcium level testing. A value of less than 100 mg in 24 hours is diagnostic of FHH and rules out PHPT.

340
Q

Asymptomatic patients who do not meet criteria for parathyroidectomy should undergo?

A

1) Annual reassessment for the development of symptoms
2) Annual blood testing to reassess whether the magnitude of their hypercalcemia has reached levels meriting parathyroidectomy
3) Bone mineral density scanning every 1 to 2 years to evaluate for the development of osteoporosis

341
Q

Indications for Parathyroidectomy in Patients With Asymptomatic Primary Hyperparathyroidism?

A

Age: <50 years

Serum calcium: 1.0 mg/dL (0.25 mmol/L) above upper limit of normal

24-hour urinary calcium: >400 mg/day (10 mmol/day)

Renal involvement: Creatinine clearance <60 mL/min or nephrolithiasis/nephrocalcinosis

Bone mineral density: T-score

342
Q

Indications for Parathyroidectomy in Patients With Secondary Hyperparathyroidism?

A

1) SHPT refractory to medical therapy:
• Parathyroid hormone >600-800 pg/mL
• Calcium × Phosphorus product >55

2) Renal osteodystrophy
3) Calciphylaxis
4) Other retractable symptoms, including uremic pruritus, persistent anemia, bone pain, muscle pain, abdominal pain, fatigue, and weakness

343
Q

Indications for Parathyroidectomy in Patients With Tertiary Hyperparathyroidism?

A

Severe hypercalcemia (calcium >12.5 mg/dL) Persistent hypercalcemia ≥2 years after renal transplantation, associated with:

  • Decline in renal function, without graft rejection
  • Nephrolithiasis
  • Progressive bone disease
  • Pancreatitis
344
Q

The Bethesda System for Reporting Thyroid Cytopathology: Implied Risk of Malignancy?

A

I Nondiagnostic or unsatisfactory (1-4% risk of malignancy)

II Benign (0-3% risk of malignancy)

III Atypia of undetermined significance or follicular lesion of undetermined significance (5-15% risk of malignancy)

IV Follicular neoplasm or suspicious for a follicular neoplasm (15-30% risk of malignancy)

V Suspicious for malignancy (60-75% risk of malignancy)

VI Malignant (97-99% risk of malignancy)

345
Q

Dalrymple sign?

A

Widened palpebral fissure

346
Q

Von Graefe sign?

A

Lid lag

347
Q

Stellwag sign?

A

Infrequent blinking

348
Q

Mean sign?

A

Global lag during elevation

349
Q

Rosenbach sign?

A

Tremor of the closed eyelids

350
Q

Griffith sign?

A

Lag of the lower lids during elevation of the globes

351
Q

Boston sign?

A

Jerking of the lagging lid

352
Q

Joffroy sign?

A

Absence of forehead wrinkling with upward gaze